SlideShare una empresa de Scribd logo
1 de 146
COMMON RESPIRATORY AND
CARDIOVASCULAR
DISEASES IN CHILDREN
Pikria Zhvania, MD Phd
CASE 1
A previously healthy 18-month-old girl was brought to the emergency
department with a 2 h history of fever and stridor. On arrival, the patient’s
temperature was 38.5°C, heart rate 200 beats/min, respiratory rate 40
breaths/min and oxygen saturation was 97% on room air. On initial
assessment, the patient had a hoarse cry, inspiratory stridor and mild
suprasternal in drawing but was able to swallow without difficulty. No
drooling was evident. There was no previous history of choking, foreign
body aspiration or sick contact. All routine immunizations, including the
18-month booster, were up to date. The preliminary diagnosis was
laryngotracheobronchitis, and the patient received nebulized budesonide
and epinephrine, followed by intravenous dexamethasone (0.6 mg/kg) and
a fluid bolus of 0.9% normal saline. A complete blood count showed a white
blood cell count of 13.9×109/L (40% polymorphonuclear cells, 2.5% band
forms), hemoglobin 118 g/L and platelets 271×109/L. A blood culture and
viral nasopharyngeal swab were performed.
The child’s stridor diminished, but drooling developed, and further
CASE 2
A 1-year-old boy presents with a 2-month history of "wet" coughing.
Over the past 3 to 6 months, he has been treated each month for
acute otitis media. His parents are concerned that, despite a good
appetite, their son has been losing weight and has four to six loose,
foul-smelling stools per day.
CASE 3
A previously healthy 2-year-old girl presents with the complaint of
acute-onset wheezing. Her mother denies previous wheezing episodes
and denies a family history of asthma or atopy. The mother says that she
left the child playing in her older brother’s room. Approximately 20
minutes later she heard the child coughing and wheezing.
Differential ds
1. allergy anaphylactic reaction?
2. foreign body obstruct
3. Bronch asthma
FUNCTIONS OF THE RESPIRATORY
SYSTEM
Ventilation:
- Movement of air into and out of the lungs
- Inspiratory phase
- Expiratory phase
Diffusion and Perfusion:
- Gas exchange across the alveolar-pulmonary
capillary membranes
Control of Breathing:
- Influenced by neural and chemical factors
- Pons, medulla, chemoreceptors in the carotid
body
- Stimulus for breathing
- Increased carbon dioxide
Hypoventilation
 Slow, shallow breathing
 Causes CO2 to build up in the blood
–Acidosi
Hyperventilation
 Rapid, deep breathing
 Causes CO2 to be blown off -
Alkalosis
Airway resistance:
Full term Newborm Airway – 1 mm edema, the
d will be 44% of normal
In adult 1 mm edema - the d will be 81% of
normal
ANATOMICAL DIFFERENCES CONT
 Chest
- More rounded at birth, A:P diameter
increases to 2:1 as child growth
- children rely on abd/diaphragmatic
breathing and are obligatory nose
breathers
 Alveoli
-9 times more at the age 12 than at birht
- fewer alveoli = less surface for gas
exchange
ASSESSMENT - HISTORY
1. Chief Complaint and HPI
Cough
Shortness of breath/Dyspnea
2. Past Health History
3. disease or breathing problems
4. Frequent severe colds, asthma, emphysema, bronchitis, pneumonia,
tuberculosis
5. Last PPD and/or chest x-ray
6. Allergies, Medication use
7. Family History Alcohol, Drugs, Home environment, Occupational
environment
8. Travel
9. Health Promotional Activities
COUGH
Onset – sudden, gradual
Duration
Nature – dry, moist, hacking, barking
Sputum – amount, color, odor
Severity – disrupts activities
Associated symptoms – sneezing, dyspnea, fever, chills, congestion,
gagging
What brings it on? – anxiety, talking, activity
What makes it better?
What has been tried? – medications, treatments
Anything similar in the past?
SHORTNESS OF BREATH (SOB) /
DYSPNEA
Onset – sudden, gradual
Duration
Severity – disrupts activities
Associated symptoms – night sweats, pain, chest pressure,
discomfort, ankle edema, diaphoresis, cyanosis
What brings it on? – position, time of day, exercise, allergens,
emotions
What makes it better?
What has been tried? – medications, inhalers, oxygen
Anything similar in the past?
DURING A BUSY NIGHT, YOU GET THE FOLLOWING
PAGE:
FYI: Sally, a 2 year old
with PNA had a desat to
88% while on 4L NC.
What do you do next? What initial management steps would you take?
HOW DO YOU INITIALLY ASSESS
A PATIENT IN RESPIRATORY
DISTRESS?
Rapid assessment
 Quickly determine severity of respiratory condition and stabilize child
 Respiratory distress can quickly lead to cardiac compromise
Airway
 Support or open airway with jaw thrust
 Suction and position patient
Breathing
 Provide high concentration oxygen
 Bag mask ventilation
 Prepare for intubation
 Administer medication ie albuterol, epinephrine
Circulation
 Establish vascular access: IV/IO
INITIAL
ASSESME
NT
HISTORY AND PHYSICAL EXAM
History
Trauma
Change in voice
Onset of symptoms
Associated symptoms
Exposures
Underlying medical conditions
Physical Exam
Mental status
Position of comfort
Nasal flaring
Accessory muscle use
Respiratory rate and pattern
Auscultation for abnormal breath
sounds
WHAT INITIAL STUDIES WOULD YOU
GET FOR A PATIENT IN
RESPIRATORY DISTRESS?
Pulse oximetry
 May be difficult in agitated patient
 May be falsely decreased in very anemic patients
Imaging
 Chest X Ray
 Consider in patients with focal lung findings or respiratory distress
of a unknown etiology
 Soft tissue radiograph of lateral neck
 May identify a retropharyngeal abscess or radiopaque foreign body
Labs
 ABG/VBG
 Chemistry: calculate anion gap
 Urine toxicology and glucose if patient has altered mental status
INITIAL STUDIES
WHAT ARE SOME EXAMPLES OF
LIFE THREATENING CONDITIONS?
Complete upper airway obstruction
No effective air movement, speech or cough
Respiratory failure
Pallor or cyanosis, altered mental status, tachypnea, bradypnea,
apnea
Tension pneumothorax
Absent breath sounds on affected side, tracheal deviation and
compromised perfusion
Pulmonary embolism
Chest pain, tachycardia, tachypnea
Cardiac tamponade
LIFE THREATENING CONDITIONS
SPECIFIC CAUSES OF
RESPIRATORY DISTRESS
Upper airway obstruction
Lower airway obstruction
Lung tissue disease
Disordered control of breathing
Upper
Airway
Disease
Croup
Foreign Body
Epiglottitis
Bacterial Tracheitis
Asthma
Bronchiolitis
Pneumonia
Foreign Body
Lower
Airway
Disease
Noise during Inspiration
Proximal to Thoracic Inlet
Nose – Pharynx – Larynx
- Awake/Crying
- it child Improves
- Nose/Pharynx
- If child Deteriorates
- Larynx
Noise during
Exhalation
Distal to Thoracic
Inlet
Trachea, Bronchi,
Causes: foreign body, tissue edema,
trauma, infection, intubation, tongue
movement to posterior pharynx with
decreased consciousness
Symptoms
Partial obstruction: noisy inspiration
(stridor), choking, gagging or vocal
changes
Complete obstruction: no audible
speech, cry or cough
UPPER AIRWAY OBSTRUCTION
LARYNGOTRACHEOBRONCHITIS -
CROUP
Viral infection (parainfluenza) with Subglottic edema; Air flow
obstruction
Affects larynx, trachea
Incidence: 6 months to 4 years, Males > Females, Fall, early
winter
Signs/Symptoms: Recurs on several nights
Cold” progressing to hoarseness, cough
Low grade fever
Night-time increase in edema with:
 Stridor
 “Seal bark” cough
 Respiratory distress
CROUP: MANAGEMENT
Mild Croup
Reassurance
Moist, cool air
Severe Croup
Humidified high concentration oxygen
Monitor EKG
IV corticosteroids per os
Nebulized racemic epinephrine
Anticipate need to intubate, assist ventilations
EPIGLOTTITIS: PATHOPHYSIOLOGY
Bacterial infection (Hemophilus influenza)
Affects epiglottis, adjacent pharyngeal tissue
Supraglottic edema
Complete Airway
Obstruction
EPIGLOTTITIS: INCIDENCE
Children > 4 years old
Common in ages 4 - 7
Pedi incidence falling due to HiB vaccination
Can occur in adults, particularly elderly
Incidence in adults is increasing
EPIGLOTTITIS: SIGNS/SYMPTOMS
Rapid onset, severe distress in hours
High fever
Intense sore throat, difficulty swallowing
Drooling
Stridor
Sits up, leans forward, extends neck slightly
One-third present unconscious, in shock
EPIGLOTTITIS: MANAGEMENT
High concentration oxygen, do not examine the throat
IV Antibiotics, if possible cefotaxime 2 mg/kg 6 h
Rapid transport
Do not attempt to visualize airway
Corticosteroids remains controversial
Immediate Life Threat
Possible Complete Airway Obstruction
CASE
A previously healthy 18-month-old girl was brought to the emergency
department with a 2 h history of fever and stridor. On arrival, the patient’s
temperature was 38.5°C, heart rate 200 beats/min, respiratory rate 40
breaths/min and oxygen saturation was 97% on room air. On initial
assessment, the patient had a hoarse cry, inspiratory stridor and mild
suprasternal indrawing but was able to swallow without difficulty. No
drooling was evident. There was no previous history of choking, foreign
body aspiration or sick contact. All routine immunizations, including the
18-month booster, were up to date. The preliminary diagnosis was
laryngotracheobronchitis, and the patient received nebulized budesonide
and epinephrine, followed by intravenous dexamethasone (0.6 mg/kg) and
a fluid bolus of 0.9% normal saline. A complete blood count showed a white
blood cell count of 13.9×109/L (40% polymorphonuclear cells, 2.5% band
forms), hemoglobin 118 g/L and platelets 271×109/L. A blood culture and
viral nasopharyngeal swab were performed.
The child’s stridor diminished, but drooling developed, and further
RETROPHARYNGEAL ABSCESS
 infection of the space between the pre-tracheal fascia and the alar fascia of
the neck and most common bacteria are :Staphylococcus and streptococcus
 most commonly occurs in children 2-4 years of age.
 Presentation:
-Toxic appearance
-generalized sore throat, fever, and discomfort
-enlarged lymph nodes of the neck and may be neck pain
-dysphasia and odynophagia “ hot potato voice”
-respiratory distress
-can also present much like a patient with peritonsillar abscess or epiglottitis
with changes in voice quality as well as drooling and posturing to maintain a
patent airway.
-Diagnosis: CBC, Blood culture, inflammatory markers, imaging studies
head in full extension
Widening
CASE
A 10-month-old presented with drooling and
neck swelling. A CT scan revealed a large
retropharyngeal abscess measuring 1.8 cm in the
anterior posterior, 6.8 cm in the lateral, and 7.5
cm in the vertical dimension (Figures 1 and 2).
The low specificity of computed tomography (CT) led investigators to try for a better way of
defining a unique set of criteria that could more effectively determine which patients require
surgical drainage.
 Treatment initially should be with broad-spectrum parenteral antibiotics.
 If retropharyngeal abscess is suspected or confirmed by CT scan, patients
should be admitted for IV antibiotics, and ENT consultation for surgical
drainage
 Usually use Ampicillin sulbactam – high doses 50-100 mg/kg +/-
Vancomicin 40 mg/kg/day or Clindamicin 25-40 mg/kg/day
 Some practitioners favor the use of glucocorticoids. However, the
evidence behind such recommendations is limited
RETROPHARYNGEAL ABSCESS
TREATMENT
Aspirated Foreign Body
Basics
Common among the 1-3 age
group who like to put everything
in their mouths
Running or falling with objects in
mouth
Inadequate chewing capabilities
Common items - gum, hot dogs,
grapes and peanuts
Presentation:
- Patients who present immediately often have severe
symptoms from the product obstructing the trachea or larynx.
- dyspnea and stridor and/or wheezing
- Another group of patients that presents within the first 72
hours are patients who have a witnessed choking episode and
an aspiration event
- 4-7 days after the aspiration might be have cough, fever,
wheezing, or stridor
Complication:
recurrent pneumonia, inflammation of the airway, and
development of atelectasis. Any pediatric patient who
presents for recurring symptoms that do not seem to improve
should be considered to have a foreign body. These foreign
bodies are almost always located in the distal airways of the
bronchi
Aspirated Foreign Body
ASPIRATED FOREIGN BODY - MANAGEMENT
if the child can cough and verbalized it is placed inthe position of comfort and
oxygen is given
IV line placement and other interventions which
may agitate
the child in this case are avoided
X-ray evaluation for localization can be
performed
urgently in stable children
The presence of asphyxia indicates the need for immediate
resuscitation and securing the airway
ILD – INTESTINAL LUNG DISEASE
Interstitial Lung Disease (ILD), also known as diffuse lung disease, includes large,
heterogeneous group of rare pulmonary disorders that cause derangements of the
alveolar wall and disordered gas exchange
Prevalence in adult – 67-81, and in infants 0.13
ChILD can be very difficult to diagnose and treat, and is associated with significant
morbidity and mortality
ChildRN Classification
• Disorders more prevalent in infancy
• Disorders of the immunocompetent host
• Disorders related to systemic disease
• Disorders of the immunocompromised host
• Disorders masquerading as ILD
PEDIATRIC ILD
CLASSIFICATION ILD
RESPIRATORY SYMPTOMS Breathlessness (most common): Initially, dyspnea on
exertion→ later at rest Nonproductive cough Pleuritic chest pain Wheeing
Hemoptysis
NON RESPIRATORY SYMPTOMS ASSOCIATED WITH DIFFERENT DPLDS • Arthritis •
Ocular • Skin and muscle • GERD • Lower GI symptoms • Recurrent sinusitis •
Neurological symptoms • Epilepsy & mental retardation • Diabetes insipidus
INVESTIGATIONS: CHEST X RAY • Typically small lung volumes with Reticular ,
Nodular, or RETICULONODULAR shadow
HIGH-RESOLUTION COMPUTED TOPOGRAPHY (HRCT) • HRCT is more sensitive •
Combinations of ground glass changes, reticulonodular shadowing, honeycomb
cysts and traction bronchiectasis
PULMONARY FUNCTION TESTING
ARTERIAL BLOOD GAS ANALYSIS • Tuberculin test
Lung biopsy- TRANS BRONCHIAL BIOPSY , OPEN LUNG BIOPSY
Treatment - Prednisone, 1 mg/kg several month, Cytotoxic agents
(Cyclophosphamide)or immunosuppressive agents (Azathioprine)
ILD – INTESTIONAL LUNG DISEASE
CYSTIC FIBROSIS- CF
 Multisystem genetic diseases affecting the lung and upper respiratory
systems, Gastrointestinal tract (GI), pancreas, liver, sweat glands.
 CF autosomal recessive disorder caused by mutation in the CF
transmembrane conductance (CFTR) protein - defective ion transport in
exocrine glands – primarily reduced CL- and secondary increased Na
absorbtion.
 Chronic bacterial infection and progressive obstructive lung disease reason
of > 90% death
 Screening: can be identified by newborn screening with elevated
immunoreactive trypsinogen (IRT) and detection of CFTR mutation
CF – CLINICAL FEATURES
Chronic recurrent Sino-pulmonary infections with CF pathogens (Staph
aureus, Pseudomonas aeruginosa). Recurrent cough, wheezing, sputum
production, obstructive lung disease
GI – Meconium ileus • Abdominal distention • Intestinal obstruction •
Increased frequency of stools • Failure to thrive (despite adequate appetite) •
Flatulence or foul-smelling flatus, steatorrhea • Recurrent abdominal pain •
Jaundice • GI bleeding
Salt-loss syndrome - hypochloremic metabolic alkalosis
Respiratory system • Cough • Recurrent wheezing • Recurrent pneumonia •
Atypical asthma • Dyspnea on exertion • Chest pain
Genitourinary tract • Undescended testicles or hydrocele • Delayed secondary
sexual development • Amenorrhea
PHYSICAL EXAMINATION
Findings related to the pulmonary system may include the following:
• Tachypnea
• Respiratory distress with retractions
• Wheeze or crackles
• Cough (dry or productive of mucoid or purulent sputum)
• Increased anteroposterior diameter of chest
• Clubbing • Cyanosis
• Hyperresonant chest upon percussion (crackles are heard acutely in
associated pneumonitis or bronchitis and chronically with bronchiectasis)
DIAGNOSIS
The diagnosis of cystic fibrosis (CF) is based on
• Typical pulmonary manifestations, GI tract
manifestations
• Family history
• Universal newborn screening
• Prenatal screening test
• Sweat test results
Requirements for a CF diagnosis include either
positive genetic testing or positive sweat chloride
test findings (>60 mEq/L) and 1 of the following:
• Typical chronic obstructive pulmonary disease
• Documented exocrine pancreatic insufficiency
• Positive family history (usually affected sibling)
Other examinations • Imaging test (X-ray, US, CT,
MRI) • Genotyping • Pulmonary function test •
Bronchoalveolar lavage and sputum microbiology •
Immunoreactive trypsinogen • Contrast barium
enema
CF MANAGEMENT
- Management The primary goals of CF treatment include the
following:
• Maintaining lung function as near to normal as possible by
controlling respiratory infection and clearing airways of mucus
• Administering nutritional therapy (ie, enzyme supplements,
multivitamin and mineral supplements) to maintain adequate growth
• Managing complications
CF MANAGEMENT
Mild acute pulmonary exacerbations of cystic fibrosis can be treated
successfully at home with the following measures:
• Increasing the frequency of airway clearance
• Inhaled bronchodilator treatment (especially if bronchial hyper
responsiveness is present or as part of airway clearance [inhaled
bronchodilator followed by chest physical therapy and postural
drainage)
• Chest physical therapy and postural drainage
• Increasing the dose of the mucolytic agent dornase alfa
(Pulmozyme)
• Use of oral antibiotics (eg, oral fluoroquinolones)
CF MANAGEMENT CONTINUE
Medications used to treat patients with cystic fibrosis may include the following:
• Pancreatic enzyme supplements
• Multivitamins (including fat-soluble vitamins
• Mucolytics
• Nebulized, inhaled, oral, or intravenous antibiotics
• Bronchodilators • Anti-inflammatory agents
• Agents to treat associated conditions or complications (eg, insulin, bisphosphonates)
• Agents devised to potentially reverse the abnormalities in chloride transport (eg,
ivacaftor)
Pancreatic enzyme supplements • Pancrelipase (Creon, Pancreaze, Ultresa, Zenpep)
Mucolytics • Dornase alpha (Pulmozyme) Bronchodilators • Inhaled beta2-agonist :
Albuterol (AccuNeb, ProAir, Proventil HFA, VoSpire ER, Ventolin HFA) Vaccination •
Vaccination against Pertussis, Haemophilus influenzae, Varicella, Streptococcus
pneumoniae, and measles and annual influenza vaccination.
Airway clearance • Postural drainage, percussion, vibration, and assisted
coughing are recommended at the time of diagnosis and should be done on
a regular basis Antibiotics (oral, intravenous, or inhalation) • Aerosolized
form : gentamicin, aztreonam, colistin, and preservative-free high-dose
tobramycin especially formulated for inhalation
Vitamins • Fat soluble vitamins A, D, E, and K and water soluble biotin, folic
acid, niacin, pantothenic acid, B vitamins (ie, B-1, B-2, B- 6, B-12), and
vitamin C. CFTR Potentiators • Cystic fibrosis transmembrane conductance
regulator (CFTR) potentiators are the first available treatment that targets
the defective CFTR protein. • Ivacaftor (Kalydeco) - facilitates increased
chloride transport by potentiating the channel-open probability (or gating)
of certain CFTR gene mutations.
CF MANAGEMENT CONTINUE
MOST COMMON CARDIAC
DISEASES IN CHILDREN
THE PATIENT
9 year boy previously healthy, football player
1 week of fatigue and dry cough
- Loss of appetite, vague stomach ache
He was seen by his doctor 2 days earlier
- Diagnosed with viral illness
2 days later he came to the ER
- Chest pain, dizzy, short of breath
NO heart problem in the family
EPIDEMIOLOGY OF HF IN
CHILDREN
In the United states
- HF related hospitalizations occus in 11.000-14.000
children yearly
- Overall mortality: 7%!!!
Congenital heart disease (69.3%)
Cardiomyopathy (13.6%)
Myocarditis (2.1%)
Arrhythmias (15.2%)
DEFINITION OF PEDIATRIC HEART
FAILURE
Inability of the heart to supply sufficient cardiac output to
keep up with metabolic demand
HEART FAILURE SYMPTOMS
Quite variable:
Swelling
Dyspnea of exertion
Palpitations
Weight gain or weight loss
Abdominal pain
Syncope
Failure to thrive
Sudden cardiac arrest
CARDIOMYOPATHY
A myocardial disorder in which heart muscle is structurally
and functionally abnormal in the absence of coronary artery
disease, hypertension, valvular disease and congenital heart
disease.
3 type:
- Dilated cardiomyopathy DCM
- Hypertrophic cardiomyopathy HCM
- Restrictive cardiomyopathy RCM
DILATED CARDIOMYOPATHY DCM
Ventricular dilatation and impaired
systolic function
Most common:>70% of pediatric
cardiomyopathy
Incidence: 0.6-1/100000
Histology: - fibrosis, myocyte
hypertrophy
CAUSES OF
DCM IN
CHILDREN
DIAGNOSIS DCM
Clinical presentations:
-Symptomatic HF (Syncope, dyspnea, overload)
-Asymptomatic
Physical examination
-Variable degrees of cardiac enlargement
-Pulse pressure is narrow
-JVP raised
-3th and 4 th sounds are common
-Mitral and tricuspid regurgitation are common
INITIAL EVALUATION OF NEW
ONSET DCM
Ecg, Echo, cardiac
XRAY
4 limb blood pressure
CBC with differencial
Serum electrolites:
magnesium, calcium,
potassium
BUN, Creatinine
UA
Liver enzymes and albumin
Thyroid tests
Cardiac catheterization
Carnitine and profile
Pyruvate lactate
Genetic evoluation
IEM: Urine for amino serum
acids
Serum free fatty acids
ECG - DCM
TREATMENT OF DCM
1. therapeutic strategy include preload and afterload reduction therapy:
Diuretics, angiotensin-converting-enzyme inhibitor and β-blockers:
Carvedilol
2. Cardiac transplantation is treatment of choice
HYPERTROPHIC CM - HCM
HCM - hypertrophied, nondilated ventricle in
the absence of a hemodynamic cause that is
capable of producing the existent magnitude of
wall thickening, excluding both physiological
hypertrophy (ie, secondary to physical activity)
and pathological hypertrophy (ie, secondary to
hypertension, aortic valvular stenosis, and
other disorders)
The primary diagnostic criterion for HCM is the
maximum diastolic septal or LV free wall
thickness >15mm or >2 standard deviation for
age and sex
Systolic function typically preserved
Abnormal cardiac relaxation
Most common cause of sudden death in
athletes
HCM CLINICAL PRESENTATION
Is difficult to diagnosed, commonly asymptomatic
 Clinical presentation
-Dyspnea, fatigue, chest pain, syncope, palpitation
-Dyspnea on effort
-Syncope on effort
-Jerky pulse
-double impulse in apex
-Pansystolic murmur
-Sudden death!!! Usually secondary to abnormal heart rhythm
HCM - INITIAL EVALUATION
1. Xray – cardiomegaly
2. ECG: changes are
nonspecific, may be
associated with abnormal
QT
3. Echo:
TREATMENT HCM
• B-Blockers and rate limiting calcium antagonists, digitalis and diuretics
are not usually beneficial
•No pharmacological treatment is known to improve prognosis
•Arrhythmia is common and responds well to amiodarone, with low
myocardial contractility
•Outflow tract obstruction can be improved by surgery
•Implantable cardiac defibrillation (ICD) for patient with risk of sudden
death
RESTRICTIVE CM
- abnormal relaxation of the
heart muscle with normal
ventricular size and thickness
- 2.5 -5% of total paediatric CM
- may be hereditary, thus
genetic screening is important
- Cause is unknown in children
- Presenting symptoms:
dyspnea, exacerbated by
respiratory illness, syncope.
Xray, Echo, Biopsy, Genetic
testing, MRI
OUR CASE
He was diagnosed with myocarditis due to
adenovirus, he had a mild improvement.
… BUT
2month later he could not walk 1 block
without having chest pain
ECHO show enlarged heart with poor
squeeze
He was re admitted and started on IV
Milrinone and diuretics.
He was gene positive, but no family
members
4 month after listing he received a heart
transplantation
He is now doing well and playing football
NEW POTENTIAL TREATMENT
1. Stem cell therapy to repair ventricular myocardium
2. Gene Therapy using a virus vector to help with abnormal
remodeling and calcium cycling
3. Surgical – better and smaller VADS
SYNCOPE
 Syncope is a sudden, brief loss of consciousness; loss of postural tone &
recovery is spontaneous.
 Up to 15 percent of children experience a syncopal episode prior to the end
of adolescence.
 Preceding symptoms: lightheadedness, nausea, visual disturbances and
palpitation.
Etiology:
- most often benign. can also occur as the result of more serious (usually
cardiac) disease with the potential of sudden death.
- Hypoglycemia (Type 1 DM)
- Supraventricular tachycardia (Rare).
- Bradycardia
- Pregnancy
COMPLAINS
fainted…
Following exercise..
Hot weather..
Light-headed..
Seeing black..
Seeing stars..
Nausea..
Abdominal cramps..
Racing heart..
Rapid standing from sitting
down..
Seeing blood..
Blood draws..
Urination..
Turning head rapidly..
Deep emotions..
Forgot what happened..
Headache..
Less common: Palpitation..
Brief convulsions.. Brief
confusion
COMMON CONDITION
COMMON CONDITIONS
Vasovagal syncope (neurocardiogenic) is the most common cause of
syncope among children ~50%.
Breath holding spells; typically occur in children between 6 months
and 24 months old; 2 types pallid and cyanotic.
Orthostatic hypotension .
Toxic exposure (opiates, alcohol, carbon monoxide ).
Medications (barbiturates, tricyclic antidepressants, and
phenothiazines)
CONDITIONS THAT MIMIC
SYNCOPES
Seizures — a seizure typically includes loss of consciousness and
postural tone.
Migraine syndromes (Basilar migraine)
Hysteria/conversion disorder.
Hyperventilation.
Choking game.
RED FLAGS HISTORY
Red flags in history
During exercise (not following exercise)
Preceding chest pain
Resuscitation required (CPR)
Family history of sudden death
Deafness
Progressive symptoms: more frequent fainting, more prolonged episodes.
Preceding palpitation.
CONGENITAL HEART DISEASES
DEFINITION
1. obstructive congenital heart lesions , for example congenital mitral
and aortic stenosis
2. Congenital heart lesions that INCREASE pulmonary arterial blood
flow = left to right shunts within the heart , for example atrial septal
defect
3. congenital heart lesions that DECREASE pulmonary arterial blood
flow – can involve right to left shunting of blood leading to systemic
cyonosis - for example tetralogy of Fallot
CONGENITAL HEART DISEASES
ACYANOTIC CONGENITAL HEART DISEASE
Left-to-Right Shunt Lesions
Atrial Septal Defect (ASD)
Ventricular Septal Defect (VSD)
Atrioventricular Septal Defect (AV Canal)
Patent Ductus Arteriosus (PDA)
Cardiac
case
Interventio
ns
Age
Color
Exa
m
Tests
CASE 1 - 9 DAY OLD. POOR FEEDING. CENTRAL
CYANOSIS
9 day
old boy
Interventions
Age
Less than 1 month
Color
Blue – Central
cyanosis
Exam
- Pulmonary N
- Pulse Ox: 75%
- Hr 160’
- Murmurs right
- No pulse delay
Tests
- CXR& ECG
- Oxygen
- Prostaglandi
n
- Fluid
balance
Differential:
 Trauma
 Sepsis
 Infection
CASE 2 - 3 WEEK OLD. POOR PERFUSION,
GREY
21 day
old boy
Interventions
Age
Less than 1 month
Color
Grey and
shock
Exam
- Pulmonary N
- Pulse Ox: 75%,
right hand 92%
- Hr 160’
- Murmurs
- pulse delay
Tests
- CXR& ECG
- Oxygen
- Prostaglandi
n
- Fluid
balance
Differential:
 Trauma
 Sepsis
 Infection
SHOCK AND LEFT SIDE
OBSTRUCTION (CLUES AND
GOALS)
 shock from cardiac obstruction or not?
Is oxygen safe? Yes mostly
PGE – trials in neonates
Fluids 10 ml/kg aliquots
 CXR – Cardiomegaly
Bedside echo
ATRIAL SEPTAL DEFECT
ASD is an opening in the atrial septum permitting free
communication of blood between the atria. Seen in 10% of
all CHD.
ATRIAL SEPTAL DEFECT
There are 3 major types:
Secundum ASD – at the Fossa Ovalis, most
common.
• Primum ASD – lower in position & is a
form of ASVD, MV cleft.
• Sinus Venosus ASD – high in the atrial
septum, associated w/partial anomalous
venous return & the least common.
ATRIAL SEPTAL DEFECT
Secundum ASD
Sinus Venosus ASD
ATRIAL SEPTAL DEFECT
Clinical Signs & Symptoms
Rarely presents with signs of CHF or other
cardiovascular symptoms.
• Most are asymptomatic but may have easy
fatigability or mild growth failure.
• Cyanosis does not occur unless pulmonary
HTN is present.
ATRIAL SEPTAL DEFECT
Clinical Signs & Symptoms
• Hyperactive precordium, RV heave, fixed widely
split S2.
• II-III/VI systolic ejection murmur @ LSB.
• Mid-diastolic murmur heard over LLSB.
ATRIAL SEPTAL DEFECT
Treatment:
Surgical or catherization laboratory closure is generally
recommended for secundum ASD w/ a Qp:Qs ratio
>2:1.
• Closure is performed electively between ages 2 &
5 yrs to avoid late complications.
• Surgical correction is done earlier in children w/
CHF or significant Pulm HTN.
ATRIAL SEPTAL DEFECT
Treatment
• Once pulmonary HTN w/ shunt reversal occurs this is
considered too late.
• Mortality is < 1%.
VENTRICULAR SEPTAL DEFECT
VSD – is an abnormal opening in the ventricular
septum, which allows free communication between the
Rt & Lt ventricles. Accounts for 25% of CHD.
VENTRICULAR SEPTAL
DEFECT
4 Types
Perimembranous (or membranous) – Most
common.
Infundibular (subpulmonary or supracristal
VSD) – involves the RV outflow tract.
• Muscular VSD – can be single or multiple.
• AVSD – inlet VSD, almost always involves
AV valvular abnormalities.
VENTRICULAR SEPTAL DEFECT
Hemodynamics
The left to right shunt occurs secondary to PVR being
< SVR, not the higher pressure in the LV.
This leads to elevated RV & pulmonary pressures &
volume hypertrophy of the LA & LV.
VENTRICULAR SEPTAL
DEFECT
Clinical Signs & Symptoms
• Small - moderate VSD, 3-6mm, are
usually
asymptomatic and 50% will close
spontaneously
by age 2yrs.
• Moderate – large VSD, almost always
have
VENTRICULAR SEPTAL
DEFECT
Clinical Signs & Symptoms
• II-III/VI harsh holosystolic murmur heard along the
LSB, more prominent with small VSD, maybe absent
with a
very Large VSD.
• Prominent P2, Diastolic murmur.
• CHF, FTT, Respiratory infections, exercise intolerance
hyperactive precordium. Symptoms develop between
1 – 6
months
VENTRICULAR SEPTAL DEFECT
Treatment
• Small VSD - no surgical intervention, no
physical restrictions, just reassurance and
periodic follow-up and endocarditis prophylaxis.
• Symptomatic VSD - Medical treatment
initially with afterload reducers & diuretics.
VENTRICULAR SEPTAL
DEFECT
Treatment
Indications for Surgical Closure:
Large VSD w/ medically uncontrolled symptomatology &
continued FTT.
Ages 6-12 mo w/ large VSD & Pulm. HTN
Age > 24 mo w/ Qp:Qs ratio > 2:1.
Supracristal VSD of any size, secondary to risk of developing
AV insufficiency.
ATRIOVENTRICULAR SEPTAL
DEFECT
AVSD results from incomplete fusion the the endocardial cushions,
which help to form the lower portion of the atrial septum, the
membranous portion of the ventricular septum and the septal
leaflets of the triscupid and mitral valves.
They account for 4% OF ALL CHD.
ATRIOVENTRICULAR SEPTAL
DEFECT
Question:
What genetic disease is AVSD more
commonly seen in?
• Answer:
Down’s Syndrome (Trisomy 21), Seen in 20-25% of cases.
ATRIOVENTRICULAR
SEPTAL DEFECT
Complete Form
Low primum ASD
continuous with a posterior
VSD.
Cleft in both septal leaflets
of TV/MV.
Results in a large L to R
shunt at both levels.
TR/MR, Pulm HTN w/
increase in PVR.
Incomplete Form
Any one of the
components may be
present.
Most common is
primum ASD, cleft in
the MV & small VSD.
Hemodynamics are
dependent on the
lesions.
ATRIOVENTRICULAR SEPTAL
DEFECT
Complete AVSD
ATRIOVENTRICULAR SEPTAL
DEFECT Clinical Signs & Symptoms
Incomplete AVSD maybe indistinguishable
from ASD - usually asymptomatic.
Congestive heart failure in infancy.
Recurrent pulmonary infections.
Failure to thrive.
Exercise intolerance, easy fatigability.
Late cyanosis from pulmonary vascular
disease w/ R to L shunt.
ATRIOVENTRICULAR
SEPTAL DEFECT
Clinical Signs & Symptoms
Hyperactive precordium
Normal or accentuated 1st hrt sound
Wide, fixed splitting of S2
Pulmonary systolic ejection murmur
w/thrill
Holosystolic murmur @ apex w/radiation to
axilla
Mid-diastolic rumbling murmur @ LSB
ATRIOVENTRICULAR SEPTAL
DEFECT
Treatment
Surgery is always required.
Treat congestive symptoms.
Pulmonary banding maybe required in premature
infants or infants < 5 kg.
Correction is done during infancy to avoid irreversible
pulmonary vascular disease.
Mortality low w/incomplete 1-2% & as high as 5% with
complete AVSD.
PATENT DUCTUS ARTERIOSUS
PDA – Persistence of the normal fetal vessel
that joins the PA to the Aorta.
Normally closes in the 1st wk of life.
Accounts for 10% of all CHD, seen in 10% of
other congenital hrt lesions and can often play
a critical role in some lesions.
Female : Male ratio of 2:1
Often associated w/ coarctation & VSD.
PATENT DUCTUS
ARTERIOSUS
Hemodynamics
As a result of higher aortic pressure, blood
shunts L to R through the ductus from
Aorta to PA.
Extent of the shunt depends on size of the
ductus & PVR:SVR.
Small PDA, pressures in PA, RV, RA are
normal.
PATENT DUCTUS ARTERIOSUS
Hemodynamics
Large PDA, PA pressures are equal to systemic pressures. In extreme
cases 70% of CO is shunted through the ductus to pulmonary
circulation.
Leads to increased pulmonary vascular disease.
PATENT DUCTUS ARTERIOSUS
Clinical Signs & Symptoms
Small PDA’s are usually asymptomatic
Large PDA’s can result in symptoms of CHF,
growth restriction, FTT.
Bounding arterial pulses
Widened pulse pressure
Enlarged heart, prominent apical impulse
Classic continuous machinary systolic murmur
Mid-diastolic murmur at the apex
PATENT DUCTUS ARTERIOSUS
Treatment
Indomethacin, inhibitor of prostaglandin synthesis can
be used in premature infants.
PDA requires surgical or catheter closure.
Closure is required treatment heart failure & to prevent
pulmonary vascular disease.
Usually done by ligation & division or intra vascular
coil.
Mortality is < 1%
OBSTRUCTIVE HEART LESIONS
Pulmonary Stenosis
Aortic Stenosis
Coarctation of the Aorta
PULMONARY STENOSIS
Pulmonary Stenosis is obstruction in the region of
either the pulmonary valve or the subpulmonary
ventricular outflow tract.
Accounts for 7-10% of all CHD.
Most cases are isolated lesions
Maybe biscuspid or fusion of 2 or more leaflets.
Can present w/or w/o an intact ventricular
septum.
PULMONARY STENOSIS
Hemodynamics
RV pressure hypertrophy  RV failure.
RV pressures maybe > systemic pressure.
Post-stenotic dilation of main PA.
W/intact septum & severe stenosis  R-L shunt through PFO 
cyanosis.
Cyanosis is indicative of Critical PS.
PULMONARY STENOSIS
Clinical Signs & Symptoms
Depends on the severity of obstruction.
Asymptomatic w/ mild PS < 30mmHg.
Mod-severe: 30-60mmHg, > 60mmHg
Prominent jugular a-wave, RV lift
Split 2nd hrt sound w/ a delay
Ejection click, followed by systolic murmur.
Heart failure & cyanosis seen in severe cases.
PULMONARY STENOSIS
Treatment
Mild PS no intervention required, close follow-up.
Mod-severe – require relieve of stenosis.
Balloon valvuloplasty, treatment of choice.
Surgical valvotomy is also a consideration.
AORTIC STENOSIS
Aortic Stenosis is an obstruction to the outflow from
the left ventricle at or near the aortic valve that causes
a systolic pressure gradient of more than 10mmHg.
Accounts for 7% of CHD.
3 Types
Valvular – Most common.
Subvalvular(subaortic) – involves the left outflow tract.
Supravalvular – involves the ascending aorta is the
least common.
AORTIC STENOSIS
Hemodynamics
Pressure hypertrophy of the LV and LA with obstruction to flow from
the LV.
Mild AS 0-25mmHG
Moderate AS 25-50mmHg
Severe AS 50-75mmHg
Critical AS > 75mmHg
AORTIC STENOSIS
Clinical Signs & Symptoms
Mild AS may present with exercise intolerance, easy
fatigabiltity, but usually asymptomatic.
Moderate AS – Chest pain, dypsnea on exertion,
dizziness & syncope.
Severe AS – Weak pulses, left sided heart failure,
Sudden Death.
AORTIC STENOSIS
Clinical Signs & Symptoms
LV thrust at the Apex.
Systolic thrill @ rt base/suprasternal notch.
Ejection click, III-IV/VI systolic murmur @ RSB/LSB w/ radiation to the
carotids.
AORTIC STENOSIS
Treatment
Because surgery does not offer a cure it is
reserved for patients with symptoms and a
resting gradient of 60-80mmHg.
For subaortic stenosis it is reserved for
gradients of 40-50mmHg because of it’s
rapidly progressive nature.
Balloon valvuloplasty is the standard of
treatment.
AORTIC STENOSIS
Treatment
Aortic insufficiency & re-stenosis is likely after surgery
and may require valve replacement.
Activity should not be restricted in Mild AS.
Mod-severe AS, no competitive sports.
COARCTATION OF THE AORTA
Coarctation- is narrowing of the aorta at varying
points anywhere from the transverse arch to the iliac
bifurcation.
98% of coarctations are juxtaductal
Male: Female ratio 3:1.
Accounts for 7 % of all CHD.
COARCTATION OF THE AORTA
Hemodynamics
Obstruction of left ventricular outflow  pressure hypertrophy of the
LV.
COARCTATION OF THE AORTA
Clinical Signs & Symptoms
Classic signs of coarctation are diminution or
absence of femoral pulses.
Higher BP in the upper extremities as compared to
the lower extremities.
90% have systolic hypertension of the upper
extremities.
Pulse discrepancy between rt & lt arms.
COARCTATION OF THE AORTA
Clinical Signs & Symptoms
With severe coarc. LE hypoperfusion, acidosis, HF and
shock.
Differential cyanosis if ductus is still open
II/VI systolic ejection murmur @ LSB.
Cardiomegaly, rib notching on X-ray.
COARCTATION OF THE AORTA
COARCTATION OF THE AORTA
Treatment
With severe coarctation maintaining the ductus with
prostaglandin E is essential.
Surgical intervention, to prevent LV dysfunction.
Angioplasty is used by some centers.
Re-coarctation can occur, balloon angioplasty is the
procedure of choice.
TOF – TETRADE FALLOT
4 anatomic malformations:
-Right Ventricular
Hypertrophy
-Pulmonary Valve
Stenosis
-Transposition of
the aorta
-Ventricular Septal
Defect
TOF
RVH
-secondary to PA Stenosis
-Increased P on RV leads to RVH
Transposition of Aorta
-aorta is displaced
VSD
-”hole in the heart”
-mixing of oxygenated and unoxygenated blood
-cyanosis
PVS
-more severe, less blood transported to the lungs and
more deoxygenated blood will pass through VSD to aorta
to be circulated throughout the body
CLINICAL PRESENTATION TOF
Clinical presentation is directly related to the degree of pulmonary stenosis.
Severe stenosis results in immediate cyanosis following birth. Mild stenosis will not
present until later.
Growth is retarded – insufficient oxygen and nutrients
SOA on exertion
SYMPTOMS:
▫ Severe cyanosis
▫ Hypercyanotic spells
- Associated with irritability or inconsolable crying
because of the hypoxia and Breathlessness and pallor
due to acidosis
▫ Squatting on exercise.
SIGNS: ▫ Clubbing in older groups ▫ Loud harsh ejection systolic murmur which will
shorten as RV outflow increases.
“TET SPELL”
“Tet spells” at 2-
3yo, child
becomes cyanotic,
may experience
syncope
MYOCARDITIS
Etiology – viral and non viral condition (autoimmune, Lyme/..)
Presentation: nonspecific GI and respiratory symptoms
 Heart failure signs in kids:
-Hepatomegaly
-Dyspnea
-Persistent Tachycardia
 Evaluation:
-ECG - Sinus tachycardia most common
-CXR – abnormal 50%
-Troponin
-BNP
MYOCARDITIS
Echocardiography is the most cost-effective test used for evaluation of
myocardial function. It is sensitive but not specific.
Findings include the following:
Global hypokinesis (the most common finding)
Increased left ventricular end diastolic and systolic dimensions
Left ventricular dysfunction, primarily systolic with decreased ejection
fraction and shortening fraction
Segmental wall motion abnormalities
Pericardial effusion
on ECG:
nonspecific ST/T wave changes and low voltages
Pseudoinfarction patterns with pathologic Q waves and poor progression
of R waves in the precordial leads may also be present. T-wave flattening
or inversion is a common finding associated with small or absent Q waves
in V5 and V6.
Case
A 15-year-old boy without cardiovascular risk factors or
previous history of cardiovascular disease presented to the
emergency department in our institution for persistent
chest pain with mild fever (<38°C) for the last 3 days. The
patient reported no respiratory tract signs.
The physical examination revealed blood pressure of
100/60 mmHg, heart rate of 75 b.p.m., oxygen saturation
of 98% while breathing ambient air, and body temperature
of 36.9°C. The electrocardiogram showed diffuse ST
elevation without reciprocal changes
Blood tests revealed a slight increase in C-reactive protein
level (41 mg/L, normal <6 mg/L) with normal leucocytes
(6.1 × 109/L, normal 4–10 × 109 cells/L) and elevated
cardiac troponin 6.1 μg/L (99th upper reference limit
0.045 μg/L). N-terminal probrain natriuretic peptide (NT-
proBNP 65 ng/L, normal <300 ng/L) and D-dimer (259
ng/mL, normal <500 ng/mL) remained normal.
Because of systematic suspicion of COVID-19 in patients
with unexplained fever, a PCR was performed on a
nasopharyngeal swab and resulted positive for SARS-CoV-
2. A multiplex real-time PCR was also performed and
resulted negative, allowing the exclusion of viral co-
infections. Chest CT scan showed no lung anomalies
Transthoracic echocardiography showed a mild diffuse
hypokinesia with left ventricular ejection fraction (LVEF) at
50%, preserved cardiac output, normal right ventricular
function, no significant valvular disease, and normal
pulmonary pressure without lower vena cava dilatation.
There was a mild pericardial effusion around the lateral
wall of the left ventricle (maximum, 5 mm) without signs of
tamponade
QUESTIONS
Examination of a 3-hr old infant reveals dysmorphic
features and cyanosis. Both the occiput and facial
profile are flat, and the fontanelle is abnormally
enlarged. The space between the great and second
toe is wide, and there is a palmar crease extending
across the left palm. Room air oximetry reveals a
saturation 70%.
QUESTIONS
Of the following, the MOST likely lesion to
be found on echocardiography would be
A. Atrioventricular septal defect
B. Coarctation of the aorta
C. Hypoplastic left heart
D. Total anomalous pulmonary venous return
E. Truncus arteriosus
QUESTIONS
After a few days of poor feeding and tachypnea, a 3 week old presents
with hypotension, poor central and peripheral pulses, and severe
metabolic acidosis. A
gallop is audible, and the heart appears enlarged on chest radiography.
hepatomegaly
is marked.
QUESTIONS
Of the following, the BEST intervention to
produce a sustained improvement is
A. 100% Oxygen administration
B. Dopamine infusion
C. Gamma globulin infusion
D. Phenylephrine infusion
E. Prostaglandin E infusion

Más contenido relacionado

Similar a Lecture_17,21 Common Respiratory and cardiac diseases in children.pptx

respiratorydistress-presentation.ppt
respiratorydistress-presentation.pptrespiratorydistress-presentation.ppt
respiratorydistress-presentation.ppt
AderawAlemie
 
Congenital Diaphragmatic Hernia.pptx
Congenital Diaphragmatic Hernia.pptxCongenital Diaphragmatic Hernia.pptx
Congenital Diaphragmatic Hernia.pptx
PradeepJoshua4
 

Similar a Lecture_17,21 Common Respiratory and cardiac diseases in children.pptx (20)

Learning points emergency resuscitation
Learning points emergency resuscitationLearning points emergency resuscitation
Learning points emergency resuscitation
 
pneumonia .pptx
pneumonia .pptxpneumonia .pptx
pneumonia .pptx
 
respiratorydistress-presentation.ppt
respiratorydistress-presentation.pptrespiratorydistress-presentation.ppt
respiratorydistress-presentation.ppt
 
Drs. Olson’s and Jackson’s CMC Pediatric X-Ray Mastery: May Cases
Drs. Olson’s and Jackson’s CMC Pediatric X-Ray Mastery: May CasesDrs. Olson’s and Jackson’s CMC Pediatric X-Ray Mastery: May Cases
Drs. Olson’s and Jackson’s CMC Pediatric X-Ray Mastery: May Cases
 
Approach to uawo
Approach to uawoApproach to uawo
Approach to uawo
 
UOUVP0hXjgghgcgjcuttfffutfdrtdduytffrdfxrgxrgd
UOUVP0hXjgghgcgjcuttfffutfdrtdduytffrdfxrgxrgdUOUVP0hXjgghgcgjcuttfffutfdrtdduytffrdfxrgxrgd
UOUVP0hXjgghgcgjcuttfffutfdrtdduytffrdfxrgxrgd
 
Respiratory Disorders ىاغفقثصضهععا(1).pdf
Respiratory Disorders ىاغفقثصضهععا(1).pdfRespiratory Disorders ىاغفقثصضهععا(1).pdf
Respiratory Disorders ىاغفقثصضهععا(1).pdf
 
Bronchiolitis final 1
Bronchiolitis final 1Bronchiolitis final 1
Bronchiolitis final 1
 
Approach to respiratory distress in children
Approach to respiratory distress in childrenApproach to respiratory distress in children
Approach to respiratory distress in children
 
Clinical Case Study: Pneumonia
Clinical Case Study: Pneumonia Clinical Case Study: Pneumonia
Clinical Case Study: Pneumonia
 
The old man with fever & acute cough
The old man with fever & acute cough The old man with fever & acute cough
The old man with fever & acute cough
 
Pals 2017 part 3
Pals 2017  part 3Pals 2017  part 3
Pals 2017 part 3
 
Near drowning
Near drowningNear drowning
Near drowning
 
Pneumonia in peadiatrics
Pneumonia in peadiatricsPneumonia in peadiatrics
Pneumonia in peadiatrics
 
Upper respiratory disorders and nursing mangement
Upper respiratory disorders and nursing mangementUpper respiratory disorders and nursing mangement
Upper respiratory disorders and nursing mangement
 
4 laryngeal disorders
4 laryngeal disorders4 laryngeal disorders
4 laryngeal disorders
 
Congenital Diaphragmatic Hernia.pptx
Congenital Diaphragmatic Hernia.pptxCongenital Diaphragmatic Hernia.pptx
Congenital Diaphragmatic Hernia.pptx
 
cough-161117122251 (1).pdf
cough-161117122251 (1).pdfcough-161117122251 (1).pdf
cough-161117122251 (1).pdf
 
Cough
CoughCough
Cough
 
Dr dinakar talk
Dr dinakar talkDr dinakar talk
Dr dinakar talk
 

Más de TorprojectTor

Más de TorprojectTor (15)

pcoss.pptx all india institute of medical sciences
pcoss.pptx all india institute of medical sciencespcoss.pptx all india institute of medical sciences
pcoss.pptx all india institute of medical sciences
 
pathophysiologyofasthma-150721074100-lva1-app6891 (2).pptx
pathophysiologyofasthma-150721074100-lva1-app6891 (2).pptxpathophysiologyofasthma-150721074100-lva1-app6891 (2).pptx
pathophysiologyofasthma-150721074100-lva1-app6891 (2).pptx
 
Casesesophagus.pptx
Casesesophagus.pptxCasesesophagus.pptx
Casesesophagus.pptx
 
Case study for spodyloarthropathies by Abhinav Dogra.pptx
Case study for spodyloarthropathies by Abhinav Dogra.pptxCase study for spodyloarthropathies by Abhinav Dogra.pptx
Case study for spodyloarthropathies by Abhinav Dogra.pptx
 
Lecture_13,14 Vasculitis and JIA.pptx
Lecture_13,14 Vasculitis and JIA.pptxLecture_13,14 Vasculitis and JIA.pptx
Lecture_13,14 Vasculitis and JIA.pptx
 
Lecture_5 Fluids.pptx
Lecture_5 Fluids.pptxLecture_5 Fluids.pptx
Lecture_5 Fluids.pptx
 
01 BLS-Part-1.pptx
01 BLS-Part-1.pptx01 BLS-Part-1.pptx
01 BLS-Part-1.pptx
 
Atherosclerosis.ppt
Atherosclerosis.pptAtherosclerosis.ppt
Atherosclerosis.ppt
 
Sepsis ppt by Abhinav Dogra for Pediatrics.pptx
Sepsis ppt by Abhinav Dogra for Pediatrics.pptxSepsis ppt by Abhinav Dogra for Pediatrics.pptx
Sepsis ppt by Abhinav Dogra for Pediatrics.pptx
 
Aging Medicine-1.pptx
Aging Medicine-1.pptxAging Medicine-1.pptx
Aging Medicine-1.pptx
 
Lec3_secondary active transport UG ST.ppt
Lec3_secondary active transport UG ST.pptLec3_secondary active transport UG ST.ppt
Lec3_secondary active transport UG ST.ppt
 
02 BLS part 2.pptx
02 BLS part 2.pptx02 BLS part 2.pptx
02 BLS part 2.pptx
 
03 BLS part 3.pptx
03 BLS part 3.pptx03 BLS part 3.pptx
03 BLS part 3.pptx
 
10 ECG interpret topic 10.pptx
10 ECG interpret topic 10.pptx10 ECG interpret topic 10.pptx
10 ECG interpret topic 10.pptx
 
Melanoma and Skin Cancer.pptx
Melanoma and Skin Cancer.pptxMelanoma and Skin Cancer.pptx
Melanoma and Skin Cancer.pptx
 

Último

Top profile Call Girls In godhra [ 7014168258 ] Call Me For Genuine Models We...
Top profile Call Girls In godhra [ 7014168258 ] Call Me For Genuine Models We...Top profile Call Girls In godhra [ 7014168258 ] Call Me For Genuine Models We...
Top profile Call Girls In godhra [ 7014168258 ] Call Me For Genuine Models We...
gajnagarg
 
Top profile Call Girls In chittoor [ 7014168258 ] Call Me For Genuine Models ...
Top profile Call Girls In chittoor [ 7014168258 ] Call Me For Genuine Models ...Top profile Call Girls In chittoor [ 7014168258 ] Call Me For Genuine Models ...
Top profile Call Girls In chittoor [ 7014168258 ] Call Me For Genuine Models ...
gajnagarg
 
Gabriel_Carter_EXPOLRATIONpp.pptx........
Gabriel_Carter_EXPOLRATIONpp.pptx........Gabriel_Carter_EXPOLRATIONpp.pptx........
Gabriel_Carter_EXPOLRATIONpp.pptx........
deejay178
 
Jual obat aborsi Jakarta ( 085657271886 )Cytote pil telat bulan penggugur kan...
Jual obat aborsi Jakarta ( 085657271886 )Cytote pil telat bulan penggugur kan...Jual obat aborsi Jakarta ( 085657271886 )Cytote pil telat bulan penggugur kan...
Jual obat aborsi Jakarta ( 085657271886 )Cytote pil telat bulan penggugur kan...
ZurliaSoop
 
Top profile Call Girls In Ratnagiri [ 7014168258 ] Call Me For Genuine Models...
Top profile Call Girls In Ratnagiri [ 7014168258 ] Call Me For Genuine Models...Top profile Call Girls In Ratnagiri [ 7014168258 ] Call Me For Genuine Models...
Top profile Call Girls In Ratnagiri [ 7014168258 ] Call Me For Genuine Models...
gajnagarg
 
一比一定(购)中央昆士兰大学毕业证(CQU毕业证)成绩单学位证
一比一定(购)中央昆士兰大学毕业证(CQU毕业证)成绩单学位证一比一定(购)中央昆士兰大学毕业证(CQU毕业证)成绩单学位证
一比一定(购)中央昆士兰大学毕业证(CQU毕业证)成绩单学位证
eqaqen
 
Top profile Call Girls In bhubaneswar [ 7014168258 ] Call Me For Genuine Mode...
Top profile Call Girls In bhubaneswar [ 7014168258 ] Call Me For Genuine Mode...Top profile Call Girls In bhubaneswar [ 7014168258 ] Call Me For Genuine Mode...
Top profile Call Girls In bhubaneswar [ 7014168258 ] Call Me For Genuine Mode...
gajnagarg
 
Top profile Call Girls In Shivamogga [ 7014168258 ] Call Me For Genuine Model...
Top profile Call Girls In Shivamogga [ 7014168258 ] Call Me For Genuine Model...Top profile Call Girls In Shivamogga [ 7014168258 ] Call Me For Genuine Model...
Top profile Call Girls In Shivamogga [ 7014168258 ] Call Me For Genuine Model...
nirzagarg
 
Top profile Call Girls In daman [ 7014168258 ] Call Me For Genuine Models We ...
Top profile Call Girls In daman [ 7014168258 ] Call Me For Genuine Models We ...Top profile Call Girls In daman [ 7014168258 ] Call Me For Genuine Models We ...
Top profile Call Girls In daman [ 7014168258 ] Call Me For Genuine Models We ...
gajnagarg
 
207095666-Book-Review-on-Ignited-Minds-Final.pptx
207095666-Book-Review-on-Ignited-Minds-Final.pptx207095666-Book-Review-on-Ignited-Minds-Final.pptx
207095666-Book-Review-on-Ignited-Minds-Final.pptx
pawangadkhe786
 
Girls in Aiims Metro (delhi) call me [🔝9953056974🔝] escort service 24X7
Girls in Aiims Metro (delhi) call me [🔝9953056974🔝] escort service 24X7Girls in Aiims Metro (delhi) call me [🔝9953056974🔝] escort service 24X7
Girls in Aiims Metro (delhi) call me [🔝9953056974🔝] escort service 24X7
9953056974 Low Rate Call Girls In Saket, Delhi NCR
 

Último (20)

Top profile Call Girls In godhra [ 7014168258 ] Call Me For Genuine Models We...
Top profile Call Girls In godhra [ 7014168258 ] Call Me For Genuine Models We...Top profile Call Girls In godhra [ 7014168258 ] Call Me For Genuine Models We...
Top profile Call Girls In godhra [ 7014168258 ] Call Me For Genuine Models We...
 
Complete Curriculum Vita for Paul Warshauer
Complete Curriculum Vita for Paul WarshauerComplete Curriculum Vita for Paul Warshauer
Complete Curriculum Vita for Paul Warshauer
 
Top profile Call Girls In chittoor [ 7014168258 ] Call Me For Genuine Models ...
Top profile Call Girls In chittoor [ 7014168258 ] Call Me For Genuine Models ...Top profile Call Girls In chittoor [ 7014168258 ] Call Me For Genuine Models ...
Top profile Call Girls In chittoor [ 7014168258 ] Call Me For Genuine Models ...
 
Gabriel_Carter_EXPOLRATIONpp.pptx........
Gabriel_Carter_EXPOLRATIONpp.pptx........Gabriel_Carter_EXPOLRATIONpp.pptx........
Gabriel_Carter_EXPOLRATIONpp.pptx........
 
Jual obat aborsi Jakarta ( 085657271886 )Cytote pil telat bulan penggugur kan...
Jual obat aborsi Jakarta ( 085657271886 )Cytote pil telat bulan penggugur kan...Jual obat aborsi Jakarta ( 085657271886 )Cytote pil telat bulan penggugur kan...
Jual obat aborsi Jakarta ( 085657271886 )Cytote pil telat bulan penggugur kan...
 
Top profile Call Girls In Ratnagiri [ 7014168258 ] Call Me For Genuine Models...
Top profile Call Girls In Ratnagiri [ 7014168258 ] Call Me For Genuine Models...Top profile Call Girls In Ratnagiri [ 7014168258 ] Call Me For Genuine Models...
Top profile Call Girls In Ratnagiri [ 7014168258 ] Call Me For Genuine Models...
 
drug book file on obs. and gynae clinical pstings
drug book file on obs. and gynae clinical pstingsdrug book file on obs. and gynae clinical pstings
drug book file on obs. and gynae clinical pstings
 
一比一定(购)中央昆士兰大学毕业证(CQU毕业证)成绩单学位证
一比一定(购)中央昆士兰大学毕业证(CQU毕业证)成绩单学位证一比一定(购)中央昆士兰大学毕业证(CQU毕业证)成绩单学位证
一比一定(购)中央昆士兰大学毕业证(CQU毕业证)成绩单学位证
 
Specialize in a MSc within Biomanufacturing, and work part-time as Process En...
Specialize in a MSc within Biomanufacturing, and work part-time as Process En...Specialize in a MSc within Biomanufacturing, and work part-time as Process En...
Specialize in a MSc within Biomanufacturing, and work part-time as Process En...
 
Top profile Call Girls In bhubaneswar [ 7014168258 ] Call Me For Genuine Mode...
Top profile Call Girls In bhubaneswar [ 7014168258 ] Call Me For Genuine Mode...Top profile Call Girls In bhubaneswar [ 7014168258 ] Call Me For Genuine Mode...
Top profile Call Girls In bhubaneswar [ 7014168258 ] Call Me For Genuine Mode...
 
B.tech Civil Engineering Major Project by Deepak Kumar ppt.pdf
B.tech Civil Engineering Major Project by Deepak Kumar ppt.pdfB.tech Civil Engineering Major Project by Deepak Kumar ppt.pdf
B.tech Civil Engineering Major Project by Deepak Kumar ppt.pdf
 
7737669865 Call Girls In Ahmedabad Escort Service Available 24×7 In In Ahmedabad
7737669865 Call Girls In Ahmedabad Escort Service Available 24×7 In In Ahmedabad7737669865 Call Girls In Ahmedabad Escort Service Available 24×7 In In Ahmedabad
7737669865 Call Girls In Ahmedabad Escort Service Available 24×7 In In Ahmedabad
 
Joshua Minker Brand Exploration Sports Broadcaster .pptx
Joshua Minker Brand Exploration Sports Broadcaster .pptxJoshua Minker Brand Exploration Sports Broadcaster .pptx
Joshua Minker Brand Exploration Sports Broadcaster .pptx
 
Novo Nordisk Kalundborg. We are expanding our manufacturing hub in Kalundborg...
Novo Nordisk Kalundborg. We are expanding our manufacturing hub in Kalundborg...Novo Nordisk Kalundborg. We are expanding our manufacturing hub in Kalundborg...
Novo Nordisk Kalundborg. We are expanding our manufacturing hub in Kalundborg...
 
Top profile Call Girls In Shivamogga [ 7014168258 ] Call Me For Genuine Model...
Top profile Call Girls In Shivamogga [ 7014168258 ] Call Me For Genuine Model...Top profile Call Girls In Shivamogga [ 7014168258 ] Call Me For Genuine Model...
Top profile Call Girls In Shivamogga [ 7014168258 ] Call Me For Genuine Model...
 
Guide to a Winning Interview May 2024 for MCWN
Guide to a Winning Interview May 2024 for MCWNGuide to a Winning Interview May 2024 for MCWN
Guide to a Winning Interview May 2024 for MCWN
 
Dating Call Girls inTiruvallur { 9332606886 } VVIP NISHA Call Girls Near 5 St...
Dating Call Girls inTiruvallur { 9332606886 } VVIP NISHA Call Girls Near 5 St...Dating Call Girls inTiruvallur { 9332606886 } VVIP NISHA Call Girls Near 5 St...
Dating Call Girls inTiruvallur { 9332606886 } VVIP NISHA Call Girls Near 5 St...
 
Top profile Call Girls In daman [ 7014168258 ] Call Me For Genuine Models We ...
Top profile Call Girls In daman [ 7014168258 ] Call Me For Genuine Models We ...Top profile Call Girls In daman [ 7014168258 ] Call Me For Genuine Models We ...
Top profile Call Girls In daman [ 7014168258 ] Call Me For Genuine Models We ...
 
207095666-Book-Review-on-Ignited-Minds-Final.pptx
207095666-Book-Review-on-Ignited-Minds-Final.pptx207095666-Book-Review-on-Ignited-Minds-Final.pptx
207095666-Book-Review-on-Ignited-Minds-Final.pptx
 
Girls in Aiims Metro (delhi) call me [🔝9953056974🔝] escort service 24X7
Girls in Aiims Metro (delhi) call me [🔝9953056974🔝] escort service 24X7Girls in Aiims Metro (delhi) call me [🔝9953056974🔝] escort service 24X7
Girls in Aiims Metro (delhi) call me [🔝9953056974🔝] escort service 24X7
 

Lecture_17,21 Common Respiratory and cardiac diseases in children.pptx

  • 1. COMMON RESPIRATORY AND CARDIOVASCULAR DISEASES IN CHILDREN Pikria Zhvania, MD Phd
  • 2. CASE 1 A previously healthy 18-month-old girl was brought to the emergency department with a 2 h history of fever and stridor. On arrival, the patient’s temperature was 38.5°C, heart rate 200 beats/min, respiratory rate 40 breaths/min and oxygen saturation was 97% on room air. On initial assessment, the patient had a hoarse cry, inspiratory stridor and mild suprasternal in drawing but was able to swallow without difficulty. No drooling was evident. There was no previous history of choking, foreign body aspiration or sick contact. All routine immunizations, including the 18-month booster, were up to date. The preliminary diagnosis was laryngotracheobronchitis, and the patient received nebulized budesonide and epinephrine, followed by intravenous dexamethasone (0.6 mg/kg) and a fluid bolus of 0.9% normal saline. A complete blood count showed a white blood cell count of 13.9×109/L (40% polymorphonuclear cells, 2.5% band forms), hemoglobin 118 g/L and platelets 271×109/L. A blood culture and viral nasopharyngeal swab were performed. The child’s stridor diminished, but drooling developed, and further
  • 3. CASE 2 A 1-year-old boy presents with a 2-month history of "wet" coughing. Over the past 3 to 6 months, he has been treated each month for acute otitis media. His parents are concerned that, despite a good appetite, their son has been losing weight and has four to six loose, foul-smelling stools per day.
  • 4. CASE 3 A previously healthy 2-year-old girl presents with the complaint of acute-onset wheezing. Her mother denies previous wheezing episodes and denies a family history of asthma or atopy. The mother says that she left the child playing in her older brother’s room. Approximately 20 minutes later she heard the child coughing and wheezing. Differential ds 1. allergy anaphylactic reaction? 2. foreign body obstruct 3. Bronch asthma
  • 5. FUNCTIONS OF THE RESPIRATORY SYSTEM Ventilation: - Movement of air into and out of the lungs - Inspiratory phase - Expiratory phase Diffusion and Perfusion: - Gas exchange across the alveolar-pulmonary capillary membranes Control of Breathing: - Influenced by neural and chemical factors - Pons, medulla, chemoreceptors in the carotid body - Stimulus for breathing - Increased carbon dioxide Hypoventilation  Slow, shallow breathing  Causes CO2 to build up in the blood –Acidosi Hyperventilation  Rapid, deep breathing  Causes CO2 to be blown off - Alkalosis
  • 6. Airway resistance: Full term Newborm Airway – 1 mm edema, the d will be 44% of normal In adult 1 mm edema - the d will be 81% of normal
  • 7. ANATOMICAL DIFFERENCES CONT  Chest - More rounded at birth, A:P diameter increases to 2:1 as child growth - children rely on abd/diaphragmatic breathing and are obligatory nose breathers  Alveoli -9 times more at the age 12 than at birht - fewer alveoli = less surface for gas exchange
  • 8. ASSESSMENT - HISTORY 1. Chief Complaint and HPI Cough Shortness of breath/Dyspnea 2. Past Health History 3. disease or breathing problems 4. Frequent severe colds, asthma, emphysema, bronchitis, pneumonia, tuberculosis 5. Last PPD and/or chest x-ray 6. Allergies, Medication use 7. Family History Alcohol, Drugs, Home environment, Occupational environment 8. Travel 9. Health Promotional Activities
  • 9. COUGH Onset – sudden, gradual Duration Nature – dry, moist, hacking, barking Sputum – amount, color, odor Severity – disrupts activities Associated symptoms – sneezing, dyspnea, fever, chills, congestion, gagging What brings it on? – anxiety, talking, activity What makes it better? What has been tried? – medications, treatments Anything similar in the past?
  • 10. SHORTNESS OF BREATH (SOB) / DYSPNEA Onset – sudden, gradual Duration Severity – disrupts activities Associated symptoms – night sweats, pain, chest pressure, discomfort, ankle edema, diaphoresis, cyanosis What brings it on? – position, time of day, exercise, allergens, emotions What makes it better? What has been tried? – medications, inhalers, oxygen Anything similar in the past?
  • 11.
  • 12. DURING A BUSY NIGHT, YOU GET THE FOLLOWING PAGE: FYI: Sally, a 2 year old with PNA had a desat to 88% while on 4L NC. What do you do next? What initial management steps would you take?
  • 13. HOW DO YOU INITIALLY ASSESS A PATIENT IN RESPIRATORY DISTRESS?
  • 14. Rapid assessment  Quickly determine severity of respiratory condition and stabilize child  Respiratory distress can quickly lead to cardiac compromise Airway  Support or open airway with jaw thrust  Suction and position patient Breathing  Provide high concentration oxygen  Bag mask ventilation  Prepare for intubation  Administer medication ie albuterol, epinephrine Circulation  Establish vascular access: IV/IO INITIAL ASSESME NT
  • 15. HISTORY AND PHYSICAL EXAM History Trauma Change in voice Onset of symptoms Associated symptoms Exposures Underlying medical conditions Physical Exam Mental status Position of comfort Nasal flaring Accessory muscle use Respiratory rate and pattern Auscultation for abnormal breath sounds
  • 16. WHAT INITIAL STUDIES WOULD YOU GET FOR A PATIENT IN RESPIRATORY DISTRESS?
  • 17. Pulse oximetry  May be difficult in agitated patient  May be falsely decreased in very anemic patients Imaging  Chest X Ray  Consider in patients with focal lung findings or respiratory distress of a unknown etiology  Soft tissue radiograph of lateral neck  May identify a retropharyngeal abscess or radiopaque foreign body Labs  ABG/VBG  Chemistry: calculate anion gap  Urine toxicology and glucose if patient has altered mental status INITIAL STUDIES
  • 18. WHAT ARE SOME EXAMPLES OF LIFE THREATENING CONDITIONS?
  • 19. Complete upper airway obstruction No effective air movement, speech or cough Respiratory failure Pallor or cyanosis, altered mental status, tachypnea, bradypnea, apnea Tension pneumothorax Absent breath sounds on affected side, tracheal deviation and compromised perfusion Pulmonary embolism Chest pain, tachycardia, tachypnea Cardiac tamponade LIFE THREATENING CONDITIONS
  • 20. SPECIFIC CAUSES OF RESPIRATORY DISTRESS Upper airway obstruction Lower airway obstruction Lung tissue disease Disordered control of breathing
  • 21. Upper Airway Disease Croup Foreign Body Epiglottitis Bacterial Tracheitis Asthma Bronchiolitis Pneumonia Foreign Body Lower Airway Disease Noise during Inspiration Proximal to Thoracic Inlet Nose – Pharynx – Larynx - Awake/Crying - it child Improves - Nose/Pharynx - If child Deteriorates - Larynx Noise during Exhalation Distal to Thoracic Inlet Trachea, Bronchi,
  • 22. Causes: foreign body, tissue edema, trauma, infection, intubation, tongue movement to posterior pharynx with decreased consciousness Symptoms Partial obstruction: noisy inspiration (stridor), choking, gagging or vocal changes Complete obstruction: no audible speech, cry or cough UPPER AIRWAY OBSTRUCTION
  • 23. LARYNGOTRACHEOBRONCHITIS - CROUP Viral infection (parainfluenza) with Subglottic edema; Air flow obstruction Affects larynx, trachea Incidence: 6 months to 4 years, Males > Females, Fall, early winter Signs/Symptoms: Recurs on several nights Cold” progressing to hoarseness, cough Low grade fever Night-time increase in edema with:  Stridor  “Seal bark” cough  Respiratory distress
  • 24. CROUP: MANAGEMENT Mild Croup Reassurance Moist, cool air Severe Croup Humidified high concentration oxygen Monitor EKG IV corticosteroids per os Nebulized racemic epinephrine Anticipate need to intubate, assist ventilations
  • 25. EPIGLOTTITIS: PATHOPHYSIOLOGY Bacterial infection (Hemophilus influenza) Affects epiglottis, adjacent pharyngeal tissue Supraglottic edema Complete Airway Obstruction
  • 26. EPIGLOTTITIS: INCIDENCE Children > 4 years old Common in ages 4 - 7 Pedi incidence falling due to HiB vaccination Can occur in adults, particularly elderly Incidence in adults is increasing
  • 27. EPIGLOTTITIS: SIGNS/SYMPTOMS Rapid onset, severe distress in hours High fever Intense sore throat, difficulty swallowing Drooling Stridor Sits up, leans forward, extends neck slightly One-third present unconscious, in shock
  • 28. EPIGLOTTITIS: MANAGEMENT High concentration oxygen, do not examine the throat IV Antibiotics, if possible cefotaxime 2 mg/kg 6 h Rapid transport Do not attempt to visualize airway Corticosteroids remains controversial Immediate Life Threat Possible Complete Airway Obstruction
  • 29.
  • 30. CASE A previously healthy 18-month-old girl was brought to the emergency department with a 2 h history of fever and stridor. On arrival, the patient’s temperature was 38.5°C, heart rate 200 beats/min, respiratory rate 40 breaths/min and oxygen saturation was 97% on room air. On initial assessment, the patient had a hoarse cry, inspiratory stridor and mild suprasternal indrawing but was able to swallow without difficulty. No drooling was evident. There was no previous history of choking, foreign body aspiration or sick contact. All routine immunizations, including the 18-month booster, were up to date. The preliminary diagnosis was laryngotracheobronchitis, and the patient received nebulized budesonide and epinephrine, followed by intravenous dexamethasone (0.6 mg/kg) and a fluid bolus of 0.9% normal saline. A complete blood count showed a white blood cell count of 13.9×109/L (40% polymorphonuclear cells, 2.5% band forms), hemoglobin 118 g/L and platelets 271×109/L. A blood culture and viral nasopharyngeal swab were performed. The child’s stridor diminished, but drooling developed, and further
  • 31. RETROPHARYNGEAL ABSCESS  infection of the space between the pre-tracheal fascia and the alar fascia of the neck and most common bacteria are :Staphylococcus and streptococcus  most commonly occurs in children 2-4 years of age.  Presentation: -Toxic appearance -generalized sore throat, fever, and discomfort -enlarged lymph nodes of the neck and may be neck pain -dysphasia and odynophagia “ hot potato voice” -respiratory distress -can also present much like a patient with peritonsillar abscess or epiglottitis with changes in voice quality as well as drooling and posturing to maintain a patent airway. -Diagnosis: CBC, Blood culture, inflammatory markers, imaging studies
  • 32. head in full extension Widening
  • 33. CASE A 10-month-old presented with drooling and neck swelling. A CT scan revealed a large retropharyngeal abscess measuring 1.8 cm in the anterior posterior, 6.8 cm in the lateral, and 7.5 cm in the vertical dimension (Figures 1 and 2). The low specificity of computed tomography (CT) led investigators to try for a better way of defining a unique set of criteria that could more effectively determine which patients require surgical drainage.
  • 34.  Treatment initially should be with broad-spectrum parenteral antibiotics.  If retropharyngeal abscess is suspected or confirmed by CT scan, patients should be admitted for IV antibiotics, and ENT consultation for surgical drainage  Usually use Ampicillin sulbactam – high doses 50-100 mg/kg +/- Vancomicin 40 mg/kg/day or Clindamicin 25-40 mg/kg/day  Some practitioners favor the use of glucocorticoids. However, the evidence behind such recommendations is limited RETROPHARYNGEAL ABSCESS TREATMENT
  • 35. Aspirated Foreign Body Basics Common among the 1-3 age group who like to put everything in their mouths Running or falling with objects in mouth Inadequate chewing capabilities Common items - gum, hot dogs, grapes and peanuts
  • 36. Presentation: - Patients who present immediately often have severe symptoms from the product obstructing the trachea or larynx. - dyspnea and stridor and/or wheezing - Another group of patients that presents within the first 72 hours are patients who have a witnessed choking episode and an aspiration event - 4-7 days after the aspiration might be have cough, fever, wheezing, or stridor Complication: recurrent pneumonia, inflammation of the airway, and development of atelectasis. Any pediatric patient who presents for recurring symptoms that do not seem to improve should be considered to have a foreign body. These foreign bodies are almost always located in the distal airways of the bronchi Aspirated Foreign Body
  • 37. ASPIRATED FOREIGN BODY - MANAGEMENT if the child can cough and verbalized it is placed inthe position of comfort and oxygen is given IV line placement and other interventions which may agitate the child in this case are avoided X-ray evaluation for localization can be performed urgently in stable children The presence of asphyxia indicates the need for immediate resuscitation and securing the airway
  • 38. ILD – INTESTINAL LUNG DISEASE Interstitial Lung Disease (ILD), also known as diffuse lung disease, includes large, heterogeneous group of rare pulmonary disorders that cause derangements of the alveolar wall and disordered gas exchange Prevalence in adult – 67-81, and in infants 0.13 ChILD can be very difficult to diagnose and treat, and is associated with significant morbidity and mortality ChildRN Classification • Disorders more prevalent in infancy • Disorders of the immunocompetent host • Disorders related to systemic disease • Disorders of the immunocompromised host • Disorders masquerading as ILD
  • 41.
  • 42. RESPIRATORY SYMPTOMS Breathlessness (most common): Initially, dyspnea on exertion→ later at rest Nonproductive cough Pleuritic chest pain Wheeing Hemoptysis NON RESPIRATORY SYMPTOMS ASSOCIATED WITH DIFFERENT DPLDS • Arthritis • Ocular • Skin and muscle • GERD • Lower GI symptoms • Recurrent sinusitis • Neurological symptoms • Epilepsy & mental retardation • Diabetes insipidus INVESTIGATIONS: CHEST X RAY • Typically small lung volumes with Reticular , Nodular, or RETICULONODULAR shadow HIGH-RESOLUTION COMPUTED TOPOGRAPHY (HRCT) • HRCT is more sensitive • Combinations of ground glass changes, reticulonodular shadowing, honeycomb cysts and traction bronchiectasis PULMONARY FUNCTION TESTING ARTERIAL BLOOD GAS ANALYSIS • Tuberculin test Lung biopsy- TRANS BRONCHIAL BIOPSY , OPEN LUNG BIOPSY Treatment - Prednisone, 1 mg/kg several month, Cytotoxic agents (Cyclophosphamide)or immunosuppressive agents (Azathioprine) ILD – INTESTIONAL LUNG DISEASE
  • 43. CYSTIC FIBROSIS- CF  Multisystem genetic diseases affecting the lung and upper respiratory systems, Gastrointestinal tract (GI), pancreas, liver, sweat glands.  CF autosomal recessive disorder caused by mutation in the CF transmembrane conductance (CFTR) protein - defective ion transport in exocrine glands – primarily reduced CL- and secondary increased Na absorbtion.  Chronic bacterial infection and progressive obstructive lung disease reason of > 90% death  Screening: can be identified by newborn screening with elevated immunoreactive trypsinogen (IRT) and detection of CFTR mutation
  • 44. CF – CLINICAL FEATURES Chronic recurrent Sino-pulmonary infections with CF pathogens (Staph aureus, Pseudomonas aeruginosa). Recurrent cough, wheezing, sputum production, obstructive lung disease GI – Meconium ileus • Abdominal distention • Intestinal obstruction • Increased frequency of stools • Failure to thrive (despite adequate appetite) • Flatulence or foul-smelling flatus, steatorrhea • Recurrent abdominal pain • Jaundice • GI bleeding Salt-loss syndrome - hypochloremic metabolic alkalosis Respiratory system • Cough • Recurrent wheezing • Recurrent pneumonia • Atypical asthma • Dyspnea on exertion • Chest pain Genitourinary tract • Undescended testicles or hydrocele • Delayed secondary sexual development • Amenorrhea
  • 45. PHYSICAL EXAMINATION Findings related to the pulmonary system may include the following: • Tachypnea • Respiratory distress with retractions • Wheeze or crackles • Cough (dry or productive of mucoid or purulent sputum) • Increased anteroposterior diameter of chest • Clubbing • Cyanosis • Hyperresonant chest upon percussion (crackles are heard acutely in associated pneumonitis or bronchitis and chronically with bronchiectasis)
  • 46. DIAGNOSIS The diagnosis of cystic fibrosis (CF) is based on • Typical pulmonary manifestations, GI tract manifestations • Family history • Universal newborn screening • Prenatal screening test • Sweat test results Requirements for a CF diagnosis include either positive genetic testing or positive sweat chloride test findings (>60 mEq/L) and 1 of the following: • Typical chronic obstructive pulmonary disease • Documented exocrine pancreatic insufficiency • Positive family history (usually affected sibling) Other examinations • Imaging test (X-ray, US, CT, MRI) • Genotyping • Pulmonary function test • Bronchoalveolar lavage and sputum microbiology • Immunoreactive trypsinogen • Contrast barium enema
  • 47.
  • 48.
  • 49.
  • 50.
  • 51. CF MANAGEMENT - Management The primary goals of CF treatment include the following: • Maintaining lung function as near to normal as possible by controlling respiratory infection and clearing airways of mucus • Administering nutritional therapy (ie, enzyme supplements, multivitamin and mineral supplements) to maintain adequate growth • Managing complications
  • 52. CF MANAGEMENT Mild acute pulmonary exacerbations of cystic fibrosis can be treated successfully at home with the following measures: • Increasing the frequency of airway clearance • Inhaled bronchodilator treatment (especially if bronchial hyper responsiveness is present or as part of airway clearance [inhaled bronchodilator followed by chest physical therapy and postural drainage) • Chest physical therapy and postural drainage • Increasing the dose of the mucolytic agent dornase alfa (Pulmozyme) • Use of oral antibiotics (eg, oral fluoroquinolones)
  • 53. CF MANAGEMENT CONTINUE Medications used to treat patients with cystic fibrosis may include the following: • Pancreatic enzyme supplements • Multivitamins (including fat-soluble vitamins • Mucolytics • Nebulized, inhaled, oral, or intravenous antibiotics • Bronchodilators • Anti-inflammatory agents • Agents to treat associated conditions or complications (eg, insulin, bisphosphonates) • Agents devised to potentially reverse the abnormalities in chloride transport (eg, ivacaftor) Pancreatic enzyme supplements • Pancrelipase (Creon, Pancreaze, Ultresa, Zenpep) Mucolytics • Dornase alpha (Pulmozyme) Bronchodilators • Inhaled beta2-agonist : Albuterol (AccuNeb, ProAir, Proventil HFA, VoSpire ER, Ventolin HFA) Vaccination • Vaccination against Pertussis, Haemophilus influenzae, Varicella, Streptococcus pneumoniae, and measles and annual influenza vaccination.
  • 54. Airway clearance • Postural drainage, percussion, vibration, and assisted coughing are recommended at the time of diagnosis and should be done on a regular basis Antibiotics (oral, intravenous, or inhalation) • Aerosolized form : gentamicin, aztreonam, colistin, and preservative-free high-dose tobramycin especially formulated for inhalation Vitamins • Fat soluble vitamins A, D, E, and K and water soluble biotin, folic acid, niacin, pantothenic acid, B vitamins (ie, B-1, B-2, B- 6, B-12), and vitamin C. CFTR Potentiators • Cystic fibrosis transmembrane conductance regulator (CFTR) potentiators are the first available treatment that targets the defective CFTR protein. • Ivacaftor (Kalydeco) - facilitates increased chloride transport by potentiating the channel-open probability (or gating) of certain CFTR gene mutations. CF MANAGEMENT CONTINUE
  • 56. THE PATIENT 9 year boy previously healthy, football player 1 week of fatigue and dry cough - Loss of appetite, vague stomach ache He was seen by his doctor 2 days earlier - Diagnosed with viral illness 2 days later he came to the ER - Chest pain, dizzy, short of breath NO heart problem in the family
  • 57. EPIDEMIOLOGY OF HF IN CHILDREN In the United states - HF related hospitalizations occus in 11.000-14.000 children yearly - Overall mortality: 7%!!! Congenital heart disease (69.3%) Cardiomyopathy (13.6%) Myocarditis (2.1%) Arrhythmias (15.2%)
  • 58. DEFINITION OF PEDIATRIC HEART FAILURE Inability of the heart to supply sufficient cardiac output to keep up with metabolic demand
  • 59. HEART FAILURE SYMPTOMS Quite variable: Swelling Dyspnea of exertion Palpitations Weight gain or weight loss Abdominal pain Syncope Failure to thrive Sudden cardiac arrest
  • 60. CARDIOMYOPATHY A myocardial disorder in which heart muscle is structurally and functionally abnormal in the absence of coronary artery disease, hypertension, valvular disease and congenital heart disease. 3 type: - Dilated cardiomyopathy DCM - Hypertrophic cardiomyopathy HCM - Restrictive cardiomyopathy RCM
  • 61. DILATED CARDIOMYOPATHY DCM Ventricular dilatation and impaired systolic function Most common:>70% of pediatric cardiomyopathy Incidence: 0.6-1/100000 Histology: - fibrosis, myocyte hypertrophy
  • 63. DIAGNOSIS DCM Clinical presentations: -Symptomatic HF (Syncope, dyspnea, overload) -Asymptomatic Physical examination -Variable degrees of cardiac enlargement -Pulse pressure is narrow -JVP raised -3th and 4 th sounds are common -Mitral and tricuspid regurgitation are common
  • 64. INITIAL EVALUATION OF NEW ONSET DCM Ecg, Echo, cardiac XRAY 4 limb blood pressure CBC with differencial Serum electrolites: magnesium, calcium, potassium BUN, Creatinine UA Liver enzymes and albumin Thyroid tests Cardiac catheterization Carnitine and profile Pyruvate lactate Genetic evoluation IEM: Urine for amino serum acids Serum free fatty acids
  • 66. TREATMENT OF DCM 1. therapeutic strategy include preload and afterload reduction therapy: Diuretics, angiotensin-converting-enzyme inhibitor and β-blockers: Carvedilol 2. Cardiac transplantation is treatment of choice
  • 67. HYPERTROPHIC CM - HCM HCM - hypertrophied, nondilated ventricle in the absence of a hemodynamic cause that is capable of producing the existent magnitude of wall thickening, excluding both physiological hypertrophy (ie, secondary to physical activity) and pathological hypertrophy (ie, secondary to hypertension, aortic valvular stenosis, and other disorders) The primary diagnostic criterion for HCM is the maximum diastolic septal or LV free wall thickness >15mm or >2 standard deviation for age and sex Systolic function typically preserved Abnormal cardiac relaxation Most common cause of sudden death in athletes
  • 68. HCM CLINICAL PRESENTATION Is difficult to diagnosed, commonly asymptomatic  Clinical presentation -Dyspnea, fatigue, chest pain, syncope, palpitation -Dyspnea on effort -Syncope on effort -Jerky pulse -double impulse in apex -Pansystolic murmur -Sudden death!!! Usually secondary to abnormal heart rhythm
  • 69. HCM - INITIAL EVALUATION 1. Xray – cardiomegaly 2. ECG: changes are nonspecific, may be associated with abnormal QT 3. Echo:
  • 70. TREATMENT HCM • B-Blockers and rate limiting calcium antagonists, digitalis and diuretics are not usually beneficial •No pharmacological treatment is known to improve prognosis •Arrhythmia is common and responds well to amiodarone, with low myocardial contractility •Outflow tract obstruction can be improved by surgery •Implantable cardiac defibrillation (ICD) for patient with risk of sudden death
  • 71. RESTRICTIVE CM - abnormal relaxation of the heart muscle with normal ventricular size and thickness - 2.5 -5% of total paediatric CM - may be hereditary, thus genetic screening is important - Cause is unknown in children - Presenting symptoms: dyspnea, exacerbated by respiratory illness, syncope. Xray, Echo, Biopsy, Genetic testing, MRI
  • 72. OUR CASE He was diagnosed with myocarditis due to adenovirus, he had a mild improvement. … BUT 2month later he could not walk 1 block without having chest pain ECHO show enlarged heart with poor squeeze He was re admitted and started on IV Milrinone and diuretics. He was gene positive, but no family members 4 month after listing he received a heart transplantation He is now doing well and playing football
  • 73. NEW POTENTIAL TREATMENT 1. Stem cell therapy to repair ventricular myocardium 2. Gene Therapy using a virus vector to help with abnormal remodeling and calcium cycling 3. Surgical – better and smaller VADS
  • 74. SYNCOPE  Syncope is a sudden, brief loss of consciousness; loss of postural tone & recovery is spontaneous.  Up to 15 percent of children experience a syncopal episode prior to the end of adolescence.  Preceding symptoms: lightheadedness, nausea, visual disturbances and palpitation. Etiology: - most often benign. can also occur as the result of more serious (usually cardiac) disease with the potential of sudden death. - Hypoglycemia (Type 1 DM) - Supraventricular tachycardia (Rare). - Bradycardia - Pregnancy
  • 75.
  • 76.
  • 77. COMPLAINS fainted… Following exercise.. Hot weather.. Light-headed.. Seeing black.. Seeing stars.. Nausea.. Abdominal cramps.. Racing heart.. Rapid standing from sitting down.. Seeing blood.. Blood draws.. Urination.. Turning head rapidly.. Deep emotions.. Forgot what happened.. Headache.. Less common: Palpitation.. Brief convulsions.. Brief confusion
  • 78. COMMON CONDITION COMMON CONDITIONS Vasovagal syncope (neurocardiogenic) is the most common cause of syncope among children ~50%. Breath holding spells; typically occur in children between 6 months and 24 months old; 2 types pallid and cyanotic. Orthostatic hypotension . Toxic exposure (opiates, alcohol, carbon monoxide ). Medications (barbiturates, tricyclic antidepressants, and phenothiazines)
  • 79. CONDITIONS THAT MIMIC SYNCOPES Seizures — a seizure typically includes loss of consciousness and postural tone. Migraine syndromes (Basilar migraine) Hysteria/conversion disorder. Hyperventilation. Choking game.
  • 80. RED FLAGS HISTORY Red flags in history During exercise (not following exercise) Preceding chest pain Resuscitation required (CPR) Family history of sudden death Deafness Progressive symptoms: more frequent fainting, more prolonged episodes. Preceding palpitation.
  • 81.
  • 82.
  • 83.
  • 84.
  • 85. CONGENITAL HEART DISEASES DEFINITION 1. obstructive congenital heart lesions , for example congenital mitral and aortic stenosis 2. Congenital heart lesions that INCREASE pulmonary arterial blood flow = left to right shunts within the heart , for example atrial septal defect 3. congenital heart lesions that DECREASE pulmonary arterial blood flow – can involve right to left shunting of blood leading to systemic cyonosis - for example tetralogy of Fallot
  • 87. ACYANOTIC CONGENITAL HEART DISEASE Left-to-Right Shunt Lesions Atrial Septal Defect (ASD) Ventricular Septal Defect (VSD) Atrioventricular Septal Defect (AV Canal) Patent Ductus Arteriosus (PDA)
  • 89. CASE 1 - 9 DAY OLD. POOR FEEDING. CENTRAL CYANOSIS 9 day old boy Interventions Age Less than 1 month Color Blue – Central cyanosis Exam - Pulmonary N - Pulse Ox: 75% - Hr 160’ - Murmurs right - No pulse delay Tests - CXR& ECG - Oxygen - Prostaglandi n - Fluid balance Differential:  Trauma  Sepsis  Infection
  • 90. CASE 2 - 3 WEEK OLD. POOR PERFUSION, GREY 21 day old boy Interventions Age Less than 1 month Color Grey and shock Exam - Pulmonary N - Pulse Ox: 75%, right hand 92% - Hr 160’ - Murmurs - pulse delay Tests - CXR& ECG - Oxygen - Prostaglandi n - Fluid balance Differential:  Trauma  Sepsis  Infection
  • 91. SHOCK AND LEFT SIDE OBSTRUCTION (CLUES AND GOALS)  shock from cardiac obstruction or not? Is oxygen safe? Yes mostly PGE – trials in neonates Fluids 10 ml/kg aliquots  CXR – Cardiomegaly Bedside echo
  • 92. ATRIAL SEPTAL DEFECT ASD is an opening in the atrial septum permitting free communication of blood between the atria. Seen in 10% of all CHD.
  • 93. ATRIAL SEPTAL DEFECT There are 3 major types: Secundum ASD – at the Fossa Ovalis, most common. • Primum ASD – lower in position & is a form of ASVD, MV cleft. • Sinus Venosus ASD – high in the atrial septum, associated w/partial anomalous venous return & the least common.
  • 94. ATRIAL SEPTAL DEFECT Secundum ASD Sinus Venosus ASD
  • 95. ATRIAL SEPTAL DEFECT Clinical Signs & Symptoms Rarely presents with signs of CHF or other cardiovascular symptoms. • Most are asymptomatic but may have easy fatigability or mild growth failure. • Cyanosis does not occur unless pulmonary HTN is present.
  • 96. ATRIAL SEPTAL DEFECT Clinical Signs & Symptoms • Hyperactive precordium, RV heave, fixed widely split S2. • II-III/VI systolic ejection murmur @ LSB. • Mid-diastolic murmur heard over LLSB.
  • 97. ATRIAL SEPTAL DEFECT Treatment: Surgical or catherization laboratory closure is generally recommended for secundum ASD w/ a Qp:Qs ratio >2:1. • Closure is performed electively between ages 2 & 5 yrs to avoid late complications. • Surgical correction is done earlier in children w/ CHF or significant Pulm HTN.
  • 98. ATRIAL SEPTAL DEFECT Treatment • Once pulmonary HTN w/ shunt reversal occurs this is considered too late. • Mortality is < 1%.
  • 99. VENTRICULAR SEPTAL DEFECT VSD – is an abnormal opening in the ventricular septum, which allows free communication between the Rt & Lt ventricles. Accounts for 25% of CHD.
  • 100. VENTRICULAR SEPTAL DEFECT 4 Types Perimembranous (or membranous) – Most common. Infundibular (subpulmonary or supracristal VSD) – involves the RV outflow tract. • Muscular VSD – can be single or multiple. • AVSD – inlet VSD, almost always involves AV valvular abnormalities.
  • 101. VENTRICULAR SEPTAL DEFECT Hemodynamics The left to right shunt occurs secondary to PVR being < SVR, not the higher pressure in the LV. This leads to elevated RV & pulmonary pressures & volume hypertrophy of the LA & LV.
  • 102. VENTRICULAR SEPTAL DEFECT Clinical Signs & Symptoms • Small - moderate VSD, 3-6mm, are usually asymptomatic and 50% will close spontaneously by age 2yrs. • Moderate – large VSD, almost always have
  • 103. VENTRICULAR SEPTAL DEFECT Clinical Signs & Symptoms • II-III/VI harsh holosystolic murmur heard along the LSB, more prominent with small VSD, maybe absent with a very Large VSD. • Prominent P2, Diastolic murmur. • CHF, FTT, Respiratory infections, exercise intolerance hyperactive precordium. Symptoms develop between 1 – 6 months
  • 104. VENTRICULAR SEPTAL DEFECT Treatment • Small VSD - no surgical intervention, no physical restrictions, just reassurance and periodic follow-up and endocarditis prophylaxis. • Symptomatic VSD - Medical treatment initially with afterload reducers & diuretics.
  • 105. VENTRICULAR SEPTAL DEFECT Treatment Indications for Surgical Closure: Large VSD w/ medically uncontrolled symptomatology & continued FTT. Ages 6-12 mo w/ large VSD & Pulm. HTN Age > 24 mo w/ Qp:Qs ratio > 2:1. Supracristal VSD of any size, secondary to risk of developing AV insufficiency.
  • 106. ATRIOVENTRICULAR SEPTAL DEFECT AVSD results from incomplete fusion the the endocardial cushions, which help to form the lower portion of the atrial septum, the membranous portion of the ventricular septum and the septal leaflets of the triscupid and mitral valves. They account for 4% OF ALL CHD.
  • 107. ATRIOVENTRICULAR SEPTAL DEFECT Question: What genetic disease is AVSD more commonly seen in? • Answer: Down’s Syndrome (Trisomy 21), Seen in 20-25% of cases.
  • 108. ATRIOVENTRICULAR SEPTAL DEFECT Complete Form Low primum ASD continuous with a posterior VSD. Cleft in both septal leaflets of TV/MV. Results in a large L to R shunt at both levels. TR/MR, Pulm HTN w/ increase in PVR. Incomplete Form Any one of the components may be present. Most common is primum ASD, cleft in the MV & small VSD. Hemodynamics are dependent on the lesions.
  • 110. ATRIOVENTRICULAR SEPTAL DEFECT Clinical Signs & Symptoms Incomplete AVSD maybe indistinguishable from ASD - usually asymptomatic. Congestive heart failure in infancy. Recurrent pulmonary infections. Failure to thrive. Exercise intolerance, easy fatigability. Late cyanosis from pulmonary vascular disease w/ R to L shunt.
  • 111. ATRIOVENTRICULAR SEPTAL DEFECT Clinical Signs & Symptoms Hyperactive precordium Normal or accentuated 1st hrt sound Wide, fixed splitting of S2 Pulmonary systolic ejection murmur w/thrill Holosystolic murmur @ apex w/radiation to axilla Mid-diastolic rumbling murmur @ LSB
  • 112. ATRIOVENTRICULAR SEPTAL DEFECT Treatment Surgery is always required. Treat congestive symptoms. Pulmonary banding maybe required in premature infants or infants < 5 kg. Correction is done during infancy to avoid irreversible pulmonary vascular disease. Mortality low w/incomplete 1-2% & as high as 5% with complete AVSD.
  • 113. PATENT DUCTUS ARTERIOSUS PDA – Persistence of the normal fetal vessel that joins the PA to the Aorta. Normally closes in the 1st wk of life. Accounts for 10% of all CHD, seen in 10% of other congenital hrt lesions and can often play a critical role in some lesions. Female : Male ratio of 2:1 Often associated w/ coarctation & VSD.
  • 114. PATENT DUCTUS ARTERIOSUS Hemodynamics As a result of higher aortic pressure, blood shunts L to R through the ductus from Aorta to PA. Extent of the shunt depends on size of the ductus & PVR:SVR. Small PDA, pressures in PA, RV, RA are normal.
  • 115. PATENT DUCTUS ARTERIOSUS Hemodynamics Large PDA, PA pressures are equal to systemic pressures. In extreme cases 70% of CO is shunted through the ductus to pulmonary circulation. Leads to increased pulmonary vascular disease.
  • 116. PATENT DUCTUS ARTERIOSUS Clinical Signs & Symptoms Small PDA’s are usually asymptomatic Large PDA’s can result in symptoms of CHF, growth restriction, FTT. Bounding arterial pulses Widened pulse pressure Enlarged heart, prominent apical impulse Classic continuous machinary systolic murmur Mid-diastolic murmur at the apex
  • 117. PATENT DUCTUS ARTERIOSUS Treatment Indomethacin, inhibitor of prostaglandin synthesis can be used in premature infants. PDA requires surgical or catheter closure. Closure is required treatment heart failure & to prevent pulmonary vascular disease. Usually done by ligation & division or intra vascular coil. Mortality is < 1%
  • 118. OBSTRUCTIVE HEART LESIONS Pulmonary Stenosis Aortic Stenosis Coarctation of the Aorta
  • 119. PULMONARY STENOSIS Pulmonary Stenosis is obstruction in the region of either the pulmonary valve or the subpulmonary ventricular outflow tract. Accounts for 7-10% of all CHD. Most cases are isolated lesions Maybe biscuspid or fusion of 2 or more leaflets. Can present w/or w/o an intact ventricular septum.
  • 120. PULMONARY STENOSIS Hemodynamics RV pressure hypertrophy  RV failure. RV pressures maybe > systemic pressure. Post-stenotic dilation of main PA. W/intact septum & severe stenosis  R-L shunt through PFO  cyanosis. Cyanosis is indicative of Critical PS.
  • 121. PULMONARY STENOSIS Clinical Signs & Symptoms Depends on the severity of obstruction. Asymptomatic w/ mild PS < 30mmHg. Mod-severe: 30-60mmHg, > 60mmHg Prominent jugular a-wave, RV lift Split 2nd hrt sound w/ a delay Ejection click, followed by systolic murmur. Heart failure & cyanosis seen in severe cases.
  • 122. PULMONARY STENOSIS Treatment Mild PS no intervention required, close follow-up. Mod-severe – require relieve of stenosis. Balloon valvuloplasty, treatment of choice. Surgical valvotomy is also a consideration.
  • 123. AORTIC STENOSIS Aortic Stenosis is an obstruction to the outflow from the left ventricle at or near the aortic valve that causes a systolic pressure gradient of more than 10mmHg. Accounts for 7% of CHD. 3 Types Valvular – Most common. Subvalvular(subaortic) – involves the left outflow tract. Supravalvular – involves the ascending aorta is the least common.
  • 124. AORTIC STENOSIS Hemodynamics Pressure hypertrophy of the LV and LA with obstruction to flow from the LV. Mild AS 0-25mmHG Moderate AS 25-50mmHg Severe AS 50-75mmHg Critical AS > 75mmHg
  • 125. AORTIC STENOSIS Clinical Signs & Symptoms Mild AS may present with exercise intolerance, easy fatigabiltity, but usually asymptomatic. Moderate AS – Chest pain, dypsnea on exertion, dizziness & syncope. Severe AS – Weak pulses, left sided heart failure, Sudden Death.
  • 126. AORTIC STENOSIS Clinical Signs & Symptoms LV thrust at the Apex. Systolic thrill @ rt base/suprasternal notch. Ejection click, III-IV/VI systolic murmur @ RSB/LSB w/ radiation to the carotids.
  • 127. AORTIC STENOSIS Treatment Because surgery does not offer a cure it is reserved for patients with symptoms and a resting gradient of 60-80mmHg. For subaortic stenosis it is reserved for gradients of 40-50mmHg because of it’s rapidly progressive nature. Balloon valvuloplasty is the standard of treatment.
  • 128. AORTIC STENOSIS Treatment Aortic insufficiency & re-stenosis is likely after surgery and may require valve replacement. Activity should not be restricted in Mild AS. Mod-severe AS, no competitive sports.
  • 129. COARCTATION OF THE AORTA Coarctation- is narrowing of the aorta at varying points anywhere from the transverse arch to the iliac bifurcation. 98% of coarctations are juxtaductal Male: Female ratio 3:1. Accounts for 7 % of all CHD.
  • 130. COARCTATION OF THE AORTA Hemodynamics Obstruction of left ventricular outflow  pressure hypertrophy of the LV.
  • 131. COARCTATION OF THE AORTA Clinical Signs & Symptoms Classic signs of coarctation are diminution or absence of femoral pulses. Higher BP in the upper extremities as compared to the lower extremities. 90% have systolic hypertension of the upper extremities. Pulse discrepancy between rt & lt arms.
  • 132. COARCTATION OF THE AORTA Clinical Signs & Symptoms With severe coarc. LE hypoperfusion, acidosis, HF and shock. Differential cyanosis if ductus is still open II/VI systolic ejection murmur @ LSB. Cardiomegaly, rib notching on X-ray.
  • 134. COARCTATION OF THE AORTA Treatment With severe coarctation maintaining the ductus with prostaglandin E is essential. Surgical intervention, to prevent LV dysfunction. Angioplasty is used by some centers. Re-coarctation can occur, balloon angioplasty is the procedure of choice.
  • 135. TOF – TETRADE FALLOT 4 anatomic malformations: -Right Ventricular Hypertrophy -Pulmonary Valve Stenosis -Transposition of the aorta -Ventricular Septal Defect
  • 136. TOF RVH -secondary to PA Stenosis -Increased P on RV leads to RVH Transposition of Aorta -aorta is displaced VSD -”hole in the heart” -mixing of oxygenated and unoxygenated blood -cyanosis PVS -more severe, less blood transported to the lungs and more deoxygenated blood will pass through VSD to aorta to be circulated throughout the body
  • 137. CLINICAL PRESENTATION TOF Clinical presentation is directly related to the degree of pulmonary stenosis. Severe stenosis results in immediate cyanosis following birth. Mild stenosis will not present until later. Growth is retarded – insufficient oxygen and nutrients SOA on exertion SYMPTOMS: ▫ Severe cyanosis ▫ Hypercyanotic spells - Associated with irritability or inconsolable crying because of the hypoxia and Breathlessness and pallor due to acidosis ▫ Squatting on exercise. SIGNS: ▫ Clubbing in older groups ▫ Loud harsh ejection systolic murmur which will shorten as RV outflow increases.
  • 138. “TET SPELL” “Tet spells” at 2- 3yo, child becomes cyanotic, may experience syncope
  • 139.
  • 140. MYOCARDITIS Etiology – viral and non viral condition (autoimmune, Lyme/..) Presentation: nonspecific GI and respiratory symptoms  Heart failure signs in kids: -Hepatomegaly -Dyspnea -Persistent Tachycardia  Evaluation: -ECG - Sinus tachycardia most common -CXR – abnormal 50% -Troponin -BNP
  • 141. MYOCARDITIS Echocardiography is the most cost-effective test used for evaluation of myocardial function. It is sensitive but not specific. Findings include the following: Global hypokinesis (the most common finding) Increased left ventricular end diastolic and systolic dimensions Left ventricular dysfunction, primarily systolic with decreased ejection fraction and shortening fraction Segmental wall motion abnormalities Pericardial effusion on ECG: nonspecific ST/T wave changes and low voltages Pseudoinfarction patterns with pathologic Q waves and poor progression of R waves in the precordial leads may also be present. T-wave flattening or inversion is a common finding associated with small or absent Q waves in V5 and V6.
  • 142. Case A 15-year-old boy without cardiovascular risk factors or previous history of cardiovascular disease presented to the emergency department in our institution for persistent chest pain with mild fever (<38°C) for the last 3 days. The patient reported no respiratory tract signs. The physical examination revealed blood pressure of 100/60 mmHg, heart rate of 75 b.p.m., oxygen saturation of 98% while breathing ambient air, and body temperature of 36.9°C. The electrocardiogram showed diffuse ST elevation without reciprocal changes Blood tests revealed a slight increase in C-reactive protein level (41 mg/L, normal <6 mg/L) with normal leucocytes (6.1 × 109/L, normal 4–10 × 109 cells/L) and elevated cardiac troponin 6.1 μg/L (99th upper reference limit 0.045 μg/L). N-terminal probrain natriuretic peptide (NT- proBNP 65 ng/L, normal <300 ng/L) and D-dimer (259 ng/mL, normal <500 ng/mL) remained normal. Because of systematic suspicion of COVID-19 in patients with unexplained fever, a PCR was performed on a nasopharyngeal swab and resulted positive for SARS-CoV- 2. A multiplex real-time PCR was also performed and resulted negative, allowing the exclusion of viral co- infections. Chest CT scan showed no lung anomalies Transthoracic echocardiography showed a mild diffuse hypokinesia with left ventricular ejection fraction (LVEF) at 50%, preserved cardiac output, normal right ventricular function, no significant valvular disease, and normal pulmonary pressure without lower vena cava dilatation. There was a mild pericardial effusion around the lateral wall of the left ventricle (maximum, 5 mm) without signs of tamponade
  • 143. QUESTIONS Examination of a 3-hr old infant reveals dysmorphic features and cyanosis. Both the occiput and facial profile are flat, and the fontanelle is abnormally enlarged. The space between the great and second toe is wide, and there is a palmar crease extending across the left palm. Room air oximetry reveals a saturation 70%.
  • 144. QUESTIONS Of the following, the MOST likely lesion to be found on echocardiography would be A. Atrioventricular septal defect B. Coarctation of the aorta C. Hypoplastic left heart D. Total anomalous pulmonary venous return E. Truncus arteriosus
  • 145. QUESTIONS After a few days of poor feeding and tachypnea, a 3 week old presents with hypotension, poor central and peripheral pulses, and severe metabolic acidosis. A gallop is audible, and the heart appears enlarged on chest radiography. hepatomegaly is marked.
  • 146. QUESTIONS Of the following, the BEST intervention to produce a sustained improvement is A. 100% Oxygen administration B. Dopamine infusion C. Gamma globulin infusion D. Phenylephrine infusion E. Prostaglandin E infusion

Notas del editor

  1. Promptly assess the patient to elicit additional important information: such as the duration of the desaturation, her general appearance and mental status, other signs of respiratory distress such as tachypnea, use of accessory muscles, grunting or flaring If the patient is still hypoxic, what can you do to quickly correct the hypoxia? increase the flow of the nasal cannula OR switch to a new delivery device such as a simple face mask While nasal cannulas can deliver up to 6L/min, flow rates greater than 4L/min are irritating to the nares and therefore switching to a simple face mask may be better tolerated) Optimize the patient’s positioning and Suction away secretions, should they be present Consider a non-rebreather face mask with an increased flow or CPAP Call for backup (such as a supervising resident or hospitalist) Consider a trial of albuterol if wheezing is present Once the patient is stabilized, it may be helpful to obtain a new chest x ray to evaluate for worsening pneumonia or development of an effusion that may have contributed to her acute decompensation.
  2. The initial assessment of a child in respiratory distress should be rapid and quickly determine if patient needs emergent interventions and rule out life threatening conditions.
  3. A brief history should be collected initially which should include these important points: A more detailed history can be collected once the child is stabilized When auscultating, listen for: wheezes crackles pleural rub prolonged expiration decreased breath sounds transmitted upper airway sounds
  4. Advantages of VBG include less pain to the patient and ability to draw concurrently with other labs A normal venous pH, pCO2, and HCO3 rules out severe acid base abnormalities A venous pH of > 7.25 predicts an arterial pH of > 7.2 in 98% of cases (Conversely, a venous pH of < 7 predicts an arterial pH of < 7.2 in 98% of cases) A venous pCO2 of > 45 mm Hg is predictive of an arterial pCO2 of > 50 mm Hb Venous blood gasses do not allow adequate determination of the arterial concentration of oxgyen (paO2) and is not as useful to quantify oxygen delivery to target tissues
  5. Croup Symptoms: barking cough stridor retractions Treatment: Oral or IM dexamethasone Oxygen Keep NPO Nebulized racemic epinephrine with observation for at least 2 hours after treatment Anaphylaxis Symptoms: Stridor or wheezing Dizziness Vomiting or diarrhea Hives or facial swelling Treatment: IM/IV epinephrine Albuterol (if bronchospasm is present) Treat hypotension Diphenhydramine Ranitidine Methylprednisolone
  6. Withn